MPRE

Pataasin ang iyong marka sa homework at exams ngayon gamit ang Quizwiz!

Which of the following is an improper fee splitting arrangement? A A lawyer paying a small portion of his fee to another lawyer who had recommended him to the client B A law firm paying a portion of a fee to a former partner under a separation agreement C A partner in a law firm sharing in a contingent fee earned solely by the work of an associate in the firm D Two lawyers from two different specialty firms entering into a written agreement with the client to combine their expertise and share a fee proportionate to each lawyer's services

A A lawyer paying a small portion of his fee to another lawyer who had recommended him to the client The ABA Model Rules prohibit a lawyer from paying anyone, including another lawyer, for recommending him to a client. Furthermore, the Rules do not permit fee splitting with a referring lawyer who neither assumes responsibility for a matter nor does work on the matter. A law firm may make payments to a former partner or associate under a separation agreement or retirement agreement. Two lawyers from two different specialty firms are permitted to share a fee if: (i) the total fee is reasonable; (ii) the split is in proportion to the services performed by each lawyer, or some different proportion if each lawyer assumes joint responsibility for the matter; and (iii) the client agrees to the split in a writing that discloses the share each lawyer will receive. The partners and associates within a law firm may split legal fees, even if the fee is earned solely by the work of one person in the firm. That is the essence of practice in a law firm.

What happens to a fee pre-paid to a lawyer if the lawyer is later fired or withdraws from the case? A If the fee was a true retainer, the lawyer generally need not refund the fee, but if the fee was an advance, the lawyer must refund any unearned portion B The lawyer must refund any unearned portion of the fee, whether the fee paid was an advance or a true retainer C If the fee was an advance, the lawyer generally need not refund the fee, but if the fee was a true retainer, the lawyer must refund any unearned portion D The lawyer generally need not refund any fee already collected, whether the fee paid was an advance or a true retainer

A If the fee was a true retainer, the lawyer generally need not refund the fee, but if the fee was an advance, the lawyer must refund any unearned portion A true retainer is money that is paid solely to ensure the availability of the lawyer, and the lawyer who is fired or withdraws generally need not refund the retainer fee. In contrast, when a lawyer requires payment in advance, she must refund any unearned part of the advance if she is later fired or withdraws.

Two years ago, when a couple divorced in State A, the court awarded the wife custody of the three children and ordered the husband to pay the wife $3,000 per month in child support and alimony payments. The husband failed to make the $3,000 payments for 17 months in a row. In desperation, the wife hired a new attorney to represent her in a proceeding to collect the past due payments from the husband. State A has no law or court rule that requires the loser to pay the winner's attorneys' fees in domestic relations matters. Because the wife had no money to pay her new attorney a regular fee, the new attorney agreed to do the work on a contingent fee basis for 10% of whatever amount the wife was ultimately able to recover. The new attorney won an award for the wife of the entire amount due ($51,000), and by tracking down and attaching the husband's secret bank account, he got the full amount paid to the wife. He then sent the wife a bill for his share, $5,100. Is the wife's new attorney subject to discipline? A No, as long as $5,100 is a reasonable fee for the work he did. B No, because the wife had no money to pay a regular fee. C Yes, because the new attorney used a contingent fee in a domestic relations matter. D Yes, because the new attorney took a portion of the money that was intended for support of the wife and the children.

A No, as long as $5,100 is a reasonable fee for the work he did. The new attorney is not subject to discipline for this fee arrangement if $5,100 is a reasonable fee. The ABA Model Rules flatly prohibit a lawyer from using a contingent fee arrangement when the payment of the fee is contingent on the securing of a divorce or an amount of alimony or support (or property settlement in lieu thereof). The Rules do not, however, prohibit a lawyer from using a contingent fee to recover money that is past due under a child support order. [ABA Model Rule 1.5(d)(1) and comment 6] In the wife's case, she had already obtained her divorce, and the amount of alimony and child support payments had already been set. The only problem was extracting the money from the husband; thus, the new attorney's use of the contingent fee arrangement in this case was proper. The contingent fee arrangement is particularly appropriate in light of the wife's lack of money to pay a regular fee and State A's failure to provide for fee shifting in domestic relations matters. [See Restatement �35, comment b] (B) is wrong because it ignores the possibility that $5,100 may be unreasonably high for the work the new attorney did. Also, the new attorney would not necessarily be subject to discipline for using this fee arrangement even if the wife had money to pay a regular fee. (C) is wrong because the collection of past due amounts of child support on a contingency fee basis is not considered a prohibited contingent fee in a domestic relations case under the Rules. (D) is wrong because it invokes a nonexistent policy. Contingent fees are generally allowed, even though they typically involve taking a share of money awarded for the support or compensation of the client (as in the ordinary personal injury case).

A bank and trust company maintains a list of approved estate and trust lawyers as a service to their customers who seek advice on estate planning matters. When a young lawyer opened her trust and estate practice in town, she asked other lawyers how she could get on the bank's approved list. They explained that the bank lists lawyers who always name the bank in wills and trust agreements they draft for clients who need an institutional executor or trustee. The bank is one of the most stable and reputable banks in the state, and its fees for executor and trustee services are competitive with those of similar institutions. In light of what she has been told by the other lawyers, may the young lawyer seek to have her name included on the bank's list? A No, because a tacit condition of being on the list is always to name the bank as executor or trustee. B No, because a lawyer must not solicit business through an intermediary. C Yes, because naming the bank causes no harm to clients who need an institutional executor or trustee. D Yes, because those who use the bank's list are already bank customers.

A No, because a tacit condition of being on the list is always to name the bank as executor or trustee. The young lawyer may not seek to have her name included on the list because naming the bank as executor or trustee in wills and trusts is a tacit condition of being on the list. A lawyer generally may not give anything of value to a person for recommending the lawyer's services. [ABA Model Rule 7.2(b)] When a lawyer names a bank as institutional executor or trustee for a client, the lawyer confers a monetary benefit on the bank in the form of the fees the bank will earn from the client's trust or estate. Here, the bank apparently lists only those lawyers who are willing to compensate it in this manner. That makes the arrangement "exclusive" and prevents the arrangement from being a reciprocal referral agreement of the kind permitted by ABA Model Rule 7.2(b)(4). Moreover, the bank's scheme creates a conflict of interest between the lawyer and the client who needs an institutional executor or trustee. The lawyer has a personal interest in staying on the bank's referral list, and that interest may skew the lawyer's judgment in advising a client whether to name the bank or a different institution. [See ABA Model Rule 1.7(a)] (B) is wrong because it is overbroad and does not hit on the specific problem with the bank's scheme. In proper circumstances, a lawyer may solicit business through an intermediary, e.g., through a prepaid legal service program or an approved lawyer referral service. (C) is wrong because a client is entitled to the unbiased advice of a lawyer in deciding what institutional trustee or executor to name. Even if the bank is just as good and its fees are as reasonable as other institutional fiduciaries, the bank's scheme deprives clients of unbiased advice. (D) is wrong because clients who use the bank for their routine banking needs might nevertheless desire to name some other institution as their executor or trustee; in any event, they are entitled to unbiased legal advice on that subject.

A potential client consulted an attorney, hoping to hire her to represent him as plaintiff in a medical malpractice action against his doctor. Without mentioning the doctor's name, the client described the alleged acts of malpractice and said that they happened more than two years ago. Only at that point did the potential client mention his doctor's name. The attorney immediately stopped the potential client and said she could not represent him because she was already representing the doctor in an unrelated matter, and she urged him to consult another lawyer. That was the end of the conversation. The potential client did nothing further for 15 months, at which point he consulted another lawyer. By that time, the statute of limitations had run on the potential client's claim against the doctor. The potential client then sued the first attorney for legal malpractice, alleging that the attorney was negligent in not warning him about the statute of limitations. Is the attorney subject to civil liability in the potential client's malpractice case? A No, because the attorney did what a reasonably prudent lawyer would do in the circumstances—decline to represent the potential client and suggest that he consult other counsel. B No, because the potential client never became the attorney's client and is therefore not a proper plaintiff in a malpractice action against the attorney. C Yes, because the attorney had no legal or ethical reason to reject the potential client as a client and therefore had a duty to warn him about the statute of limitations. D Yes, because a reasonably prudent lawyer would have foreseen that the potential client might delay in consulting another lawyer.

A No, because the attorney did what a reasonably prudent lawyer would do in the circumstances—decline to represent the potential client and suggest that he consult other counsel. Generally speaking, a lawyer must not represent a client in a presently pending piece of litigation and simultaneously oppose that client in a different piece of litigation, without each client's informed consent, confirmed in writing. [See comment 6 to ABA Model Rule 1.7] Here, the attorney may have believed that the conflict of interest that would have been created by his undertaking representation of the prospective client was unconsentable. Thus, he acted properly in declining to represent the prospective client, even though the prospective client's case was unrelated to the one in which the attorney was representing the doctor. However, a lawyer does owe a duty of reasonable care to a prospective client, even though no attorney-client relationship ever comes about. [See Restatement of the Law Governing Lawyers (hereinafter "Restatement") �15] Ordinarily, that duty would include cautioning the prospective client about an impending statute of limitations deadline. [Id.] Here, however, a cautionary word to the prospective client would constitute disloyalty to the existing client, the doctor. [See, e.g., Flatt v. Superior Court, 9 Cal. 4th 275 (1994)—warning prospective client about statute of limitations was not required when it would be disloyal to present client] The attorney therefore acted properly in simply suggesting that the prospective client consult other counsel. (B) is incorrect because a lawyer does owe a duty of reasonable care to a prospective client. (C) is incorrect because the attorney did have a legal and ethical reason to reject the prospective client as a client: the simultaneous representation of the doctor in the unrelated matter. (D) is incorrect because the foreseeability of the harm is not the whole of the analysis. Even if the attorney could foresee that the prospective client would dawdle and let the statute of limitations run, the attorney's duty of loyalty to the doctor required him not to warn the prospective client about the statute of limitations.

An insurance company offers a legal services insurance policy. In return for a yearly premium, an insured will be reimbursed by the insurance company for a specified amount for legal services during the year. The insured selects a lawyer from a list of "authorized providers" supplied by the insurance company. Any lawyer who agrees to follow a maximum fee schedule set by the insurance company can become an "authorized provider." The insurance company solicits insurance sales by in-person and live telephone contact with potential insurance buyers, working systematically through local telephone directories. Will an attorney be subject to discipline if he becomes an "authorized provider" and receives clients through the insurance company's insurance plan? A No, because the insurance company does not specifically target persons whom it knows are in need of legal services in a particular matter covered by its insurance plan. B No, because the insurance company's insureds are allowed to select whatever lawyer they wish from among the "authorized providers." C Yes, because the insurance company uses a specified maximum fee schedule. D Yes, because the insurance company uses in-person and live telephone solicitation to get business.

A No, because the insurance company does not specifically target persons whom it knows are in need of legal services in a particular matter covered by its insurance plan. The attorney will not be subject to discipline if he becomes an authorized provider under the insurance company's plan. The insurance company has set up a prepaid legal services plan of the kind referred to in the ABA Model Rules. A lawyer may receive legal business through such a plan, unless the operator of the plan uses live person-to-person contact to solicit people who it knows are in need of legal services in a particular matter covered by the plan. [ABA Model Rule 7.3(e)] (B) is wrong because there is no rule concerning the selection of counsel in a legal services insurance plan. (C) is also wrong because it would be an antitrust violation (and also an ethics violation) for a group of lawyers to conspire to follow a minimum or maximum fee schedule [see Arizona v. Maricopa County Medical Society, 457 U.S. 332 (1982)], but it is neither illegal nor unethical for a lawyer to agree to follow an insurance company's maximum fee schedule for work done for that company's insured. (D) is wrong because it is too broad. ABA Model Rule 7.3(e) permits a lawyer to participate in a legal services plan that uses live person-to-person contact, as long as it does not contact persons who are known to need legal services in a particular matter covered by the plan.

A contingent fee may be valid even if it is ________. A Not based on a percentage of the amount recovered by the client B Oral C Based on the amount of alimony awarded to the client

A Not based on a percentage of the amount recovered by the client A contingent fee may be valid even if it is not based on a percentage of the amount recovered by the client; an otherwise proper contingent fee may still be proper even if there is no res, or pool of money, from which the fee can be paid. A contingent fee may not be oral; it must be in writing signed by the client, and must state: (i) How the fee is to be calculated, including the percentage that the lawyer will get if the case is settled before trial, won after trial, or won after appeal; (ii) What litigation and other expenses are to be deducted from the recovery; (iii) Whether deductions for expenses will be made before or after the contingent fee is calculated; and (iv) What expenses the client must pay, whether or not she wins the case. A contingent fee is not permitted in criminal cases or domestic relations cases; hence, a fee based on the amount of alimony awarded to the client is not permitted.

A lawyer may properly bill a client for all of the following, EXCEPT: A Overhead expenses associated with staffing, equipping, and running the attorney's office B Travel time to and from depositions C Incidental costs such as photocopying, long distance calls, and computer research associated with the case D Services provided by third parties, such as court reporters, travel agents, and expert witnesses

A Overhead expenses associated with staffing, equipping, and running the attorney's office An attorney must disclose the basis on which a client will be charged for legal services and expenses, and the attorney's bill should clearly show how the amount due has been computed. An attorney must not charge the client for ordinary overhead expenses associated with staffing, equipping, and running the attorney's office. An attorney may charge a client for the actual cost to the attorney of special services, such as photocopying, long distance calls, computer research, etc. An attorney may charge a client for services provided by third parties, such as court reporters, travel agents, and expert witnesses, but must not charge the client more than her actual cost. An attorney may charge for travel time, such as to and from depositions, but the attorney must not "double bill" her time. For example, if she works on another client's case while on the airplane to a deposition, she may not bill both clients for both the travel and the work.

When a lawyer receives funds from a third party that are to be used, in part, to pay the lawyer's fee, the lawyer must __________. A Place the funds in a client trust account until there is an accounting and severance of the respective interests of the client and the lawyer B Place the funds into the hands of an objective third party until an accounting takes place C Place the lawyer's portion of the funds into the lawyer's business account and the client's portion of the funds into a client trust account D Place the funds in the lawyer's business fund account until there is an accounting and severance of the respective interests of the client and the lawyer

A Place the funds in a client trust account until there is an accounting and severance of the respective interests of the client and the lawyer A lawyer sometimes receives funds from a third party that are to be used, in part, to pay the lawyer's fee. The lawyer must place such funds in a client trust account until there is an accounting and severance of the respective interests of the client and the lawyer. If the client disputes the amount that is due to the lawyer, then the disputed portion must be kept in the client trust account until the dispute is resolved. The lawyer must not place any portion of the funds in the lawyer's business fund account. But there is no need for the lawyer to turn the funds over to a third party until the accounting takes place.

Which of the following statements is correct? A The attorney-client privilege protects only confidential communications, whereas the ethical duty of confidentiality applies to all information that relates to the representation of the client B Both the attorney-client privilege and the ethical duty of confidentiality apply to all information that relates to the representation of the client C Both the attorney-client privilege and the ethical duty of confidentiality protect only confidential communications D The ethical duty of confidentiality protects only confidential communications, whereas the attorney-client privilege applies to all information that relates to the representation of the client

A The attorney-client privilege protects only confidential communications, whereas the ethical duty of confidentiality applies to all information that relates to the representation of the client The ethical duty of confidentiality covers more kinds of information than the attorney-client privilege. The attorney-client privilege protects only confidential communications between the attorney and client (or the agents of either of them). The ethical duty, in contrast, covers not only confidential communications, but also any other information that the attorney obtains relating to the representation of the client, no matter what the source of that information. The ethical duty thus applies to all information that relates to the representation of the client, regardless of whether it is privileged, whether the client asked for it to be kept in confidence, and whether revealing it might harm or embarrass the client.

What is required for a communication to be deemed "confidential"? A The communicating person must reasonably believe that no outsider will hear the contents of the statement B The communicating person must reasonably believe that no outsider will hear the contents of the statement, and in fact no outsider hears the contents of the statement C The communicating person must state that the communication is to be considered confidential D The communicating person must state that the communication is to be considered confidential and in fact no outsider hears the contents of the statement

A The communicating person must reasonably believe that no outsider will hear the contents of the statement For a communication to be deemed confidential, it must have been made by a means not intended to disclose the communicated information to outsiders, and the communicating person must reasonably believe that no outsider will hear the contents of the statement. There is no requirement that the communicating person state that the communication is to be considered confidential. Furthermore, even if the communicating person does make such a request, the communication would not be confidential if knowingly made in the presence of outsiders, who are not present to help further the attorney-client relationship. Under older law, the presence of an unsuspected eavesdropper was sometimes held to destroy the confidentiality of a communication, but under modern evidence law, that is no longer true and an eavesdropper can be prohibited from testifying about a confidential communication. Thus, it is not a requirement that no outsider hears the contents of the statement.

The ultimate power to regulate the legal profession rests with __________. A The highest court in the state B The state legislature C The American Bar Association D The state bar association

A The highest court in the state Because the practice of law is intimately connected with the administration of justice, the ultimate power to regulate the legal profession in and out of court rests with the highest court in the state. The highest court generally promulgates the ethics rules and oversees the discipline of lawyers. Every state has a body of statutes passed by the state legislature that govern some aspects of the practice of law, but the ultimate power rests with the highest court in the state, not with the state legislature. Common functions of a state bar association are to administer the state's bar examination, to provide continuing education programs for practicing lawyers, and to assist the state courts in regulating and imposing professional discipline on lawyers. Nearly every state has adopted some version of the American Bar Association ("ABA") Model Rules of Professional Conduct. Likewise, most of the states have adopted some version of the ABA Model Code of Judicial Conduct, but the ABA itself does not have the power to regulate the legal profession.

What is a lawyer's duty regarding confidential information supplied by a client with diminished capacity who faces a risk of substantial physical, financial, or other harm? A The lawyer has implied authority to reveal the client's confidential information, but only to the extent necessary to protect the client B The lawyer has the discretion to use the information as he sees fit to best serve the client, because a client with diminished capacity has no expectation of privacy C The lawyer must keep the client's information confidential at all times, unless he has petitioned a court for express authority to reveal the information D The lawyer remains under an ethical duty to keep the client's information confidential at all times

A The lawyer has implied authority to reveal the client's confidential information, but only to the extent necessary to protect the client When a client has diminished capacity and faces a risk of substantial physical, financial, or other harm, the lawyer may take reasonable actions to protect the client. When taking protective action, the lawyer has implied authority to reveal the client's confidential information, but only to the extent necessary to protect the client. These actions include consulting with people or entities that can protect the client, and, when appropriate, seeking the appointment of a guardian or similar surrogate. The lawyer remains under a general ethical duty to keep the client's information confidential, but NOT at all times, such as the case here when the client is facing a risk of substantial physical, financial, or other harm. It is not necessary for the lawyer to petition a court for express authority to reveal the information, because in such a situation the lawyer has the implied authority to act as necessary to protect the client from substantial harm. It is incorrect to state that a client with diminished capacity has no expectation of privacy. The lawyer has a duty, so far as reasonably possible, to maintain a normal lawyer-client relationship with the client. The lawyer must treat the client with attention and respect.

A lawyer is a partner in a private law firm. That firm regularly provides legal services to three major banks and two other important lending institutions in the community. The lawyer has been invited to become a member of the board of directors of the local legal aid society, the group that sets overall governing policies for the local legal aid office. One of the major issues that will soon face the board of directors is whether to amend the case intake guidelines to allow the legal aid office to represent clients in disputes with banks and other lending agencies. Which of the following statements is correct? A The lawyer may join the board of directors, but she must refrain from participating in the decision about the case intake guidelines. B The lawyer will be subject to discipline if she joins the board of directors because service on the board is in conflict with the interests of her firm's bank and lending institution clients. C It would be proper for the lawyer to join the board of directors, and it would be proper for her to participate in the decision about the case intake guidelines. D The lawyer may join the board of directors to help discharge her pro bono obligation, and she may vote in favor of amending the case intake guidelines in order to make it easier for low income persons to sue banks and other lending institutions.

A The lawyer may join the board of directors, but she must refrain from participating in the decision about the case intake guidelines. The lawyer may join the board of directors, but she must refrain from participating in the decision about the case intake guidelines. A lawyer may not participate in a legal service board decision that may adversely affect one of the lawyer's clients. [ABA Model Rule 6.3(b)] (B) is wrong because ABA Model Rule 6.3 encourages work with a legal services organization, even if the organization serves people whose interests conflict with the interests of the lawyer's clients. (C) and (D) are wrong because ABA Model Rule 6.3(b) prohibits a lawyer from taking part in a legal services organization decision if the decision will adversely affect one of the lawyer's clients.

A retired lawyer practiced admiralty and maritime law for 45 years in Maine. He stopped paying his bar dues in Maine when he retired, and he is no longer licensed to practice there. He and his wife moved to a retirement village in New Mexico, but he did not seek to become licensed to practice law in New Mexico. After a few months of playing golf and puttering in the garden, the retired lawyer got bored and started missing the challenges of law practice. He therefore joined the unpaid staff of volunteer lawyers at the Rio Grande Walk-In Legal Advice Clinic, which is run by a nonprofit organization. The clinic's purpose is to offer free, quick, accurate, compassionate legal advice to walk-in clients who cannot afford ordinary legal service and who have legal problems that can be solved quickly, without litigation or other time-consuming procedures. Before they ever see one of the clinic's lawyers, all of the clients must give informed consent to the limited nature of the legal services they will receive. The retired lawyer works at the clinic three days a week, and he dispenses legal advice on all sorts of matters—although he has yet to find a client who needed admiralty or maritime advice. The retired lawyer enjoys the work because it makes him feel useful again, and because it gives him a cornucopia of interesting stories to tell his wife about his clients' various legal troubles. Which of the following statements is correct? A The retired lawyer is subject to discipline for practicing law without a license. B The retired lawyer is subject to discipline for failing to pay his bar dues in Maine. C The retired lawyer's volunteer work is proper because one does not need to be licensed to dispense legal advice at a quick-service clinic like this one. D The retired lawyer's conversations with his wife are proper because no confidential lawyer-client relationship is formed at a quick-service clinic like this one.

A The retired lawyer is subject to discipline for practicing law without a license. (A) is correct because the retired lawyer's dispensation of legal advice constitutes the "practice of law," and he is doing it in a jurisdiction where he is not licensed. [See ABA Model Rule 5.5(a)] (B) is incorrect because the retired lawyer is under no obligation to pay bar dues in a state where he no longer lives or practices. (C) is incorrect because one does need to be licensed in order to dispense legal advice at a walk-in legal clinic. [See ABA Model Rule 6.5, which loosens the conflict of interest rules for clinic lawyers but does not authorize them to practice without a license] A law student or similar unlicensed person can work at such a clinic under the close supervision of a lawyer, but the question makes no mention of the retired lawyer's work being supervised. [See comment 1 to ABA Model Rule 5.3] (D) is incorrect because there is a lawyer-client relationship between the retired lawyer and his walk-in clients. [Comment 1 to ABA Model Rule 6.5] One element of that relationship is the lawyer's duty of confidentiality [ABA Model Rule 1.6], and the retired lawyer breaches that duty when he tells his wife about his clients' legal troubles.

An attorney received her law degree two years ago from a small local college of law and technical sciences. Last summer she attended a three-day trial practice seminar at the Harvard Law School. During her brief career, she has tried five cases—two jury trials and three bench trials. She won both of the jury trials and two of the three bench trials. The attorney placed an ad under the subject heading "Trial Lawyers" in the classified pages of the local phone book. Her ad states in relevant part: "Trial Attorney Harvard Trained Never Lost a Jury Trial" Which of the following is correct? A To make the ad proper, the references to "Harvard Trained" and "Never Lost a Jury Trial" must be deleted. B To make the ad proper, the references to "Trial Attorney" and "Harvard Trained" must be deleted. C To make the ad proper, the references to "Trial Attorney" and "Never Lost a Jury Trial" must be deleted. D The ad is proper as written.

A To make the ad proper, the references to "Harvard Trained" and "Never Lost a Jury Trial" must be deleted. The attorney would be subject to discipline for the last two statements in her advertisement. It is misleading for her to state that she is "Harvard Trained," because reasonable readers could interpret that to mean that she received her law degree from that school. [See ABA Model Rule 7.1 and comment 2] The statement "Never Lost a Jury Trial," although literally true, could create unjustified expectations and is therefore misleading. [See ABA Model Rule 7.1 and comment 3] The reference to "Trial Attorney" would not make the lawyer subject to discipline. Given her brief time in law practice, she has had significant experience as a trial lawyer. A lawyer is allowed to state the fields of law in which she does or does not practice. [ABA Model Rule 7.2, comment 9] (B), (C), and (D) are all incorrect because they would permit "Harvard Trained" or "Never Lost a Jury Trial" (or both) to remain in the advertisement.

If, during the course of litigation, a lawyer discovers that a client is using her services to further an illegal course of action, the lawyer must __________. A Withdraw from the case and, depending on the circumstances, disaffirm her prior work on the matter to potentially alert the opposing side to her client's wrongdoing B Continue with the case to the best of her ability, remaining silent as to the client's impropriety so as not to violate attorney-client privilege C Reveal the details of the impropriety to the judge in chambers for advice on how to best proceed under the unique circumstances of the case D Withdraw from the case and make a full disclosure to both the court and opposing counsel of the details of her client's impropriety so as to avoid being sanctioned

A Withdraw from the case and, depending on the circumstances, disaffirm her prior work on the matter to potentially alert the opposing side to her client's wrongdoing When a lawyer discovers that a client has begun an illegal course of action and the action is continuing, the lawyer must not assist in the wrongdoing and must withdraw from the case. Withdrawal alone is often not enough—the lawyer may have to make a "noisy withdrawal" in which she gives outsiders notice of her withdrawal and disaffirms any of her own prior opinions, documents, affirmations, or the like that the client is using to carry out the wrongdoing. The lawyer's noisy withdrawal may potentially alert the opposing side, but this alone is not a violation of the attorney-client privilege. It would be a violation of the attorney-client privilege to reveal all the details of the client's impropriety to the court or opposing counsel. In such circumstances, the attorney need not seek advice from the judge on how best to proceed, but should automatically withdraw from the case as discussed above. The lawyer is not free to continue with the case if it is evident that the illegal course of action is continuing.

An attorney is representing a defendant on trial for armed robbery of a liquor store. The defendant tells the attorney in confidence that at the time in question, he was sitting at home watching television with his aged mother, and that his mother can confirm his alibi. The attorney interviews the mother, who solemnly confirms the defendant's story. After talking with her, the attorney strongly suspects that she is lying to protect the defendant. The attorney does not know for sure that the defendant and his mother are lying, but every instinct tells him that they are. The attorney has warned both of them about the dangers of perjury, but both have insisted that they want to testify to the alibi at trial. May the attorney call the defendant, or his mother, or both, as trial witnesses? A Yes, as to both the defendant and his mother. B Yes, as to the defendant, but no, as to his mother. C No, as to both the defendant and his mother. D No, as to the defendant, but yes, as to his mother.

A Yes, as to both the defendant and his mother. A lawyer has a general duty of loyalty to the client and a general duty to represent the client with dedication and commitment to the client's interest and with zeal in advocacy on the client's behalf. When acting as an advocate for a client, as the attorney is doing here, a lawyer must resolve reasonable doubts in favor of his client. There are certain situations under which the lawyer must act in a way that is adverse to his client, but each of the situations requires that the lawyer have actual knowledge of adverse facts, not just doubt or suspicion. [See ABA Model Rule 3.3(a)] Here, the attorney does not have actual knowledge that the defendant and his mother are lying or even any knowledge of circumstantial facts that indicate that they are lying; at best, he has a strong suspicion based on his instincts. This falls well short of actual knowledge. Note that a lawyer may refuse to offer evidence if he reasonably believes that it is false, other than the testimony of a criminal defendant. [ABA Model Rule 3.3(a)(3)] This rule does not apply to the defendant's testimony because the defendant is a criminal defendant; it also does not apply to the defendant's mother's testimony for two reasons: First, the question asks whether the attorney may offer the testimony, not whether he may refuse to offer it. Second, there is nothing in the facts to indicate that the attorney's belief that the testimony is false is reasonable. Thus, the attorney may call the defendant and his mother as trial witnesses.

A judge serves on a state trial court that has nine other judges. Her husband is a life insurance salesman for a large life insurance company. The life insurance company is occasionally a litigant in the court on which the judge sits. Every year the life insurance company runs a national sales contest in which the person who sells the most life insurance during the year receives a valuable prize. The judge's husband won this year and took the judge on an all-expense-paid vacation in Europe. She did not make a public report of the prize. Was it proper for the judge to allow her husband to accept the prize and take her on the European vacation? A Yes, because acceptance of the prize cannot reasonably be perceived as undermining the judge's integrity or impartiality. B Yes, because the prize was won by her husband, not by the judge. C No, because the judge did not make a public report of the prize. D No, because the life insurance company may later appear as a litigant in the court on which the judge sits.

A Yes, because acceptance of the prize cannot reasonably be perceived as undermining the judge's integrity or impartiality. It was proper for the judge to allow her husband to accept the prize because acceptance thereof does not reasonably appear to undermine the judge's integrity or impartiality. A judge may accept benefits associated with her spouse's business activity that incidentally benefit the judge. [CJC Rule 3.13(B)(8)] (B) is wrong because it ignores the general rule on family members' accepting gifts and other benefits. (C) is wrong because such benefits of a spouse are not subject to the public reporting requirement. [CJC Rule 3.13(B)] (D) is wrong because if the prize is proper under CJC Rule 3.13(B)(8), it does not become improper simply because the insurance company may later appear as a litigant in the judge's court.

A lawyer represents the defendant in a criminal case. The defendant is charged with vehicular homicide, a felony. Under the criminal statute in question, a defendant is guilty if he caused the victim's death by driving a motor vehicle either intentionally or recklessly in disregard of the safety of others. In the defendant's case, the critical issue is whether the traffic light facing the defendant's traffic lane was green at a specified moment. If the light was green, then the defendant is not guilty, but if it was red, then the defendant is guilty. The defendant himself has blocked the entire event from memory and has no idea whether the light was green or red. Five bystanders were in a position to see the light at the time in question. The lawyer interviewed four of them. With varying degrees of uncertainty, all four of them told the lawyer that they believe the light was red but that they are not positive. Based on their recollections, as well as certain physical evidence in the case, the lawyer herself believes that the light was probably red, but of course she was not present at the scene and cannot be certain. Then the lawyer interviewed the fifth bystander, who said that he simply could not remember what color the traffic light was. The lawyer replied: "My client is facing 20 years in jail, and the whole case against him turns on the color of that light. My client and I would both be eternally grateful to you if you could testify that the light was green. Would you help us out?" After thinking it over, the fifth bystander said he would be glad to help by testifying that the light was green. At the trial, the lawyer presented the fifth bystander's testimony that he saw the light, that he remembers what color it was, and that it was green. The jury believed the fifth bystander, and the defendant was acquitted. Is the lawyer subject to criminal liability for inducing the fifth bystander to testify falsely? A Yes, because both the bystander and the lawyer knew that the bystander did not remember what color the light was. B Yes, because neither the bystander nor the lawyer was certain that the light was green. C No, because neither the bystander nor the lawyer was certain what color the light was. D No, because the defense lawyer in a criminal case must resolve all doubtful facts in her client's favor when she presents evidence on her client's behalf.

A Yes, because both the bystander and the lawyer knew that the bystander did not remember what color the light was. The lawyer is subject to criminal liability because she knew that the fifth bystander did not remember the color of the light. The crime of subornation of perjury is the corrupt procurement of perjured testimony. [See R. Perkins & R. Boyce, Criminal Law (hereinafter "Perkins & Boyce") 524-26 (3d ed. 1982)] Perjured testimony means a false oath in a judicial proceeding in regard to a material matter. [Id.] A false oath means a willful and corrupt sworn statement made without sincere belief in its truthfulness. [Id.] In this case, the falsehood was not about the color of the traffic light, but rather about the fifth bystander's memory of the color of the traffic light. The fifth bystander testified that he saw the light and remembered what color it was—but in truth, he did not remember. The lawyer knew that the bystander did not remember, and the lawyer knew that the bystander knew that he did not remember. Therefore, the lawyer is guilty of subornation of perjury. (B) and (C) are both incorrect because they focus on the wrong thing—the color of the light, rather than the witness's memory. (D) is a generally correct statement, but it does not apply here. A criminal defense lawyer must indeed resolve doubtful facts in her client's favor, but the state of the bystander's memory was not a doubtful fact—the bystander told the lawyer at the outset that he "simply could not remember what color the traffic light was."

A young lawyer, three years out of law school, had never set foot in a courtroom. The lawyer was on the board of directors of a nonprofit preschool. One of the preschool's teachers was charged with felony child abuse for allegedly molesting three pupils. After conducting its own careful investigation, the preschool's board of directors concluded that the criminal charge was totally unfounded, and the board resolved to provide defense counsel for the teacher. The young lawyer volunteered to do the work without a fee. A few days before the trial was to begin, the lawyer became convinced that he was not competent to serve as the teacher's trial counsel. He asked the trial judge for permission to withdraw. After thoroughly questioning the lawyer about his preparation for trial, the judge said that while he understood the lawyer's anxiety, he believed that the lawyer was perfectly competent to handle the case. The judge denied the lawyer's motion to withdraw but postponed the trial for seven days to allow him to complete his preparation. Instead of doing what the judge ordered, the lawyer advised the teacher that he would not defend her. He handed her all of the files in the case and advised her to retain another lawyer. Is the lawyer subject to discipline? A Yes, because he abandoned his client in direct violation of the trial judge's order. B Yes, because he undertook a case that he was not competent to handle. C No, because he believed that he was not competent to represent his client at trial. D No, because he was working pro bono, not for a fee.

A Yes, because he abandoned his client in direct violation of the trial judge's order. When ordered to do so by a tribunal, a lawyer must continue representation notwithstanding good cause for terminating the representation. [ABA Model Rule 1.16(c)] Even if the lawyer was absolutely convinced of his own incompetence, he is subject to discipline for abandoning the teacher in violation of the trial judge's order. (C) is wrong for the reason stated above. (B) is wrong because a lawyer is not subject to discipline for taking on a case that he is not competent to handle if he puts in the time and study needed to make himself competent to handle it. [See comment 4 to ABA Model Rule 1.1] Having volunteered to take on the case, the lawyer's duty was to put in the requisite time and study. (D) is wrong because a lawyer's duties of competence, diligence, and loyalty are no lower in a pro bono matter than in a fee-paying matter.

A lawyer assigned his secretary to manage his client trust account. The lawyer gave the secretary extensive, detailed instructions about the kinds of records to keep, the kinds of funds that she must deposit, and the kinds of permissible withdrawals that she could make. The lawyer had complete faith in the secretary's ability and honesty, and therefore did not supervise the secretary's management of the account. Three years later, during an audit, it was discovered that on 18 different occasions during that period, the account balance fell below the amount that should have been there. The lawyer was unaware of these occasions until he received a copy of the audit. Is the lawyer subject to discipline? A Yes, because he did not adequately supervise the secretary. B Yes, because a lawyer must manage his client trust account himself. C No, because he took reasonable steps to train the secretary and did not realize that the account balance had fallen below the proper level. D No, because he did not have actual knowledge that the secretary was not performing the account management function properly.

A Yes, because he did not adequately supervise the secretary. ABA Model Rule 5.3(b) requires that "a lawyer having direct supervisory authority over the nonlawyer shall make reasonable efforts to ensure that the person's conduct is compatible with the professional obligations of the lawyer." Although the lawyer provided "extensive, detailed instructions" regarding the client trust account, he subsequently did nothing to supervise the secretary's management of the account, thereby violating Rule 5.3(b). (B) is incorrect because a lawyer is not required to manage his client trust account himself. (C) and (D) are incorrect because the lawyer's duty to supervise is not contingent on learning of a potential problem.

A full-time judge lives in State A. Her father lives in a retirement home in State B. The judge's father told her that several of his friends in the retirement home had employed an attorney to write wills for them, and that in each will the attorney had included a bequest to himself. Each bequest was approximately 50% of the estimated total value of the person's probable estate. The friends told the judge's father that they did not really want to leave the attorney anything, but they had assumed it was merely a matter of routine, a part of the attorney's compensation for drafting the will. The attorney is admitted to practice in State B, but not in State A. The judge did not talk personally with any of her father's friends, but she believes that her father's rendition of the story is entirely accurate. Would it be proper for the judge to communicate directly with the attorney about the matter, and if that does not satisfy her, to communicate with the attorney disciplinary authority in State B about the matter? A Yes, because she has received information indicating a substantial likelihood that the attorney has violated a legal ethics rule. B Yes, because she has personal knowledge that the attorney has violated a legal ethics rule. C No, because legal ethics violations that take place outside State A are not her concern. D No, because she is not allowed to communicate directly with the attorney about the supposed legal ethics violation.

A Yes, because she has received information indicating a substantial likelihood that the attorney has violated a legal ethics rule. Yes, it would be proper for the judge to take the steps mentioned because she has received information indicating a substantial likelihood that the attorney has violated a legal ethics rule. The attorney has apparently violated ABA Model Rule 1.8(c), which generally prohibits a lawyer from drafting a will under which he will receive a substantial gift. A judge who receives "information indicating a substantial likelihood" that a lawyer has violated a legal ethics rule must take "appropriate action," which may include direct communication with the lawyer, direct action, and reporting the violation to the appropriate authority. [CJC Rule 2.15(D)] (B) is wrong because it is factually incorrect—the judge does not have personal knowledge; rather she has received second-hand information about the attorney. [See CJC, Terminology] (C) is wrong because the duties imposed by CJC Rule 2.15 are not confined to lawyers in the judge's own jurisdiction. (D) is wrong because "appropriate action" may include direct communication with the lawyer who violated the legal ethics rule. [CJC Rule 2.15(D), comment 2]

After graduating from law school, an attorney was admitted to practice in one state and not in any other jurisdiction. She joined the United States Army Judge Advocate General's ("JAG") Corps—the corps of lawyer-soldiers who provide legal services to the Army throughout the world. After completing her officer training and her training in military law, she was assigned to the JAG office at a military base in a different state. Even though she was not admitted to practice in that state, she was assigned to the legal assistance desk. According to Army regulations, her job is to provide legal services to military personnel and their dependents concerning a wide range of personal legal problems, including civil, domestic, and financial matters. An officer and his wife ask the attorney for legal advice about financing a mobile home, which they plan to put in a mobile home park located in the town closest to the military base. The attorney knows absolutely nothing about the business and legal issues involved in financing a mobile home, but she is willing to undertake additional research to learn about these issues. Would it be proper for the attorney to give the requested advice to the officer and his wife? A Yes, because she is willing to do the research necessary to give competent advice on mobile home financing. B No, because she is not knowledgeable about these business and legal issues. C No, because she is not admitted to practice general civil law in the new state. D No, because mobile home financing is not directly related to the Army's mission.

A Yes, because she is willing to do the research necessary to give competent advice on mobile home financing. The attorney does not need to be admitted to practice in the new state because the JAG Corps is an organ of the federal government, and Army regulations authorize JAG officers to provide legal services to Army personnel and their dependents on a wide range of personal legal problems. [See ABA Model Rule 5.5(d)(2)] But before she advises the officer and his wife, the attorney must do enough research to become competent on the legal aspects of mobile home financing. [See comment 2 to ABA Model Rule 1.1] (B) is wrong because she is willing to undertake the additional study necessary to become competent in these matters, and therefore her current lack of knowledge is not dispositive. [See comment 2 to ABA Model Rule 1.1] (C) is wrong for the reason stated in the explanation of (A), above. (D) is wrong because the problem specifies that Army regulations authorize a legal assistance officer to give legal advice on a wide range of personal legal problems that affect military personnel and their dependents. These regulations reflect the Army's strong interest in keeping its people out of legal troubles no matter what the source.

An attorney represented a defendant in a criminal trial. After the jury returned a guilty verdict, the defendant was taken to jail and the jury was discharged. While walking to his car, the disappointed attorney spotted one of the courtroom spectators in the parking lot. The attorney recalled that the spectator had been a member of the jury pool, but he had exercised a peremptory challenge against her because he instinctively felt that she would vote against the defendant. Despite not being selected as a juror, the spectator developed an interest in the case and had attended the entire trial. In an attempt to determine whether his instinct during jury selection was correct, the attorney approached the spectator and asked her whether she would have voted to convict the defendant. The spectator said, "I'd rather not talk about it." When the attorney explained that he was simply looking for constructive feedback, the spectator changed her mind and agreed to a brief interview. The attorney and spectator spoke for a few minutes, and the communication did not involve misrepresentation, coercion, duress, or harassment. Is the attorney subject to discipline? A Yes, because the spectator initially declined to speak with the attorney. B Yes, because post-trial contact with prospective jurors is prohibited. C No, because the communication did not involve misrepresentation, coercion, duress, or harassment. D No, because the spectator was not chosen for the jury.

A Yes, because the spectator initially declined to speak with the attorney. The attorney is subject to discipline because the spectator initially declined the attorney's request for an interview. ABA Model Rule 3.5(c) provides that after the trial is over and the jury is discharged, a lawyer must not communicate with a former juror or prospective juror if any of the following conditions is met: (i) local law or a court order prohibits such communication; (ii) the juror has told the lawyer that she does not want to communicate; or (iii) the communication involves misrepresentation, coercion, duress, or harassment. Here, the attorney violated the second condition—he persisted with his interview request after the spectator said that she did not want to talk with him. (C) is incorrect. Even though the communication did not involve coercion, duress, or harassment, the attorney still spoke with the spectator after she declined his request, violating the rule. (D) is incorrect because ABA Model Rule 3.5(c) applies to all jurors and even prospective jurors. (B) is too broad. There is no blanket prohibition regarding post-trial contact with jurors and prospective jurors. Rather, such communications are subject to conditions, and the attorney violated one of these conditions.

What is considered a "communication" with respect to the attorney-client privilege? A Only information passed directly between the attorney and the client B Any information passed from the client or the client's agent to the attorney or the attorney's agent, and any information passed from the attorney or the attorney's agent to the client or the client's agent C Only information passed from the client or the client's agent to the attorney or the attorney's agent D Only information passed from the attorney or the attorney's agent to the client or the client's agent

B Any information passed from the client or the client's agent to the attorney or the attorney's agent, and any information passed from the attorney or the attorney's agent to the client or the client's agent When discussing the attorney-client privilege, the term "communication" covers information passed from the client to the attorney and from the attorney to the client. It also covers information passed to or from the agents of either the attorney or the client. Thus, the information does not have to pass directly between the attorney and the client to be covered.

A lawyer who is admitted to practice law in one state, but who improperly practices in another state, will be subject to the disciplinary rules of __________. A Only the state of her improper practice B Both the state where she is admitted and the state of her improper practice C Only the state where she is admitted D The federal court system

B Both the state where she is admitted and the state of her improper practice A lawyer who is admitted to practice law in one state is not, without more, authorized to practice in any other state. A lawyer is subject to discipline for practicing in a state where she is not admitted to practice. A lawyer who is admitted to practice law in one state, but who improperly practices in another state, will be subject to the disciplinary rules of both the state where she is admitted AND the state of her improper practice. Discipline of such a lawyer will not be handled by the federal court system.

A lawyer's assent to accept a client __________. A Must be clearly communicated to the client, either in writing or orally, to be legally binding B Can be implied when he fails to clearly decline representation and the prospective client reasonably relies on the representation C Can be implied because a lawyer has an ethical duty to take any case offered upon tender of a proper fee D Must be in writing to be legally binding under the Statute of Frauds

B Can be implied when he fails to clearly decline representation and the prospective client reasonably relies on the representation A lawyer's assent to accept a client is implied when he fails to clearly decline representation and the prospective client reasonably relies on the representation. The reasonableness of the reliance is a question of fact. It is false to state that a lawyer's assent to accept a client can be implied because a lawyer has an ethical duty to take any case offered upon tender of a proper fee. Lawyers in the United States are generally free to refuse service to any person for any reason. The fact that a prospective client tenders a proper fee to a lawyer, by itself, does not create a lawyer-client relationship. The lawyer has to agree to provide the service. As stated above, a lawyer's assent can be implied when the lawyer fails to decline the representation and the client reasonably relies on the representation; thus it is helpful, but not necessary, for the lawyer to clearly communicate his assent to the client. There is no Statute of Frauds requirement that an agreement to represent a client must be in writing to be binding.

Which of the following acts would NOT constitute an act of moral turpitude sufficient to deny admission to the bar? A Committing a crime involving intentional dishonesty for personal gain B Committing an act of nonviolent civil disobedience C Committing a violent crime D Concealing facts in response to an inquiry by the bar admissions committee

B Committing an act of nonviolent civil disobedience An act of nonviolent civil disobedience does not rise to the level of moral turpitude; hence, it would not be sufficient by itself to disqualify the candidate from admission to the bar. Crimes involving intentional dishonesty for the purpose of personal gain (e.g., forgery, bribery, theft, perjury, robbery, extortion) and crimes involving violence (e.g., murder, rape, mayhem) are generally considered to be crimes of moral turpitude. False statements or concealment of facts in response to an inquiry by the bar admissionscommittee also constitutes moral turpitude. Such concealment alone is evidence of sufficient lack of moral character to deny admission, even if the underlying conduct does not involve moral turpitude.

Which of the following acts done by a lawyer would be considered assisting another in the unauthorized practice of law? A Advising a nonlawyer on aspects of the law related to that nonlawyer's job B Having a paralegal send a routine request to opposing counsel without first checking the contents of the letter C Hiring a disbarred lawyer as a legal secretary D Giving a nonlawyer advice on how to appear pro se before a court

B Having a paralegal send a routine request to opposing counsel without first checking the contents of the letter A lawyer who assists a nonlawyer in the unauthorized practice of law is subject to professional discipline. A lawyer may delegate certain tasks to a paralegal, law clerk, student intern, or other such person, but only if the lawyer supervises the delegated work carefully and is ultimately responsible for the results. For example, a paralegal may write and sign letters on the law firm letterhead to make routine requests for information from banks, mortgage companies, and governmental agencies, provided that she indicates that she is a paralegal, not a lawyer. But a paralegal should not sign letters to clients, adversaries, opposing counsel, or tribunals; she may draft such letters, but they should be approved and signed by the lawyer. That helps assure that the lawyer is properly supervising the paralegal's work. A lawyer may advise persons who wish to appear on their own behalf in a legal matter. Thus, giving a nonlawyer advice on how to appear pro se would not be considered aiding another in the unauthorized practice of law. A lawyer may properly advise and instruct nonlawyers whose employment requires a knowledge of the law—e.g., claims adjusters, bank trust officers, social workers, accountants, and government employees. It is proper to hire a suspended or disbarred lawyer to do work that a layperson is permitted to do, such as working as a legal secretary, but the suspended or disbarred lawyer must not be permitted to do any work that constitutes the practice of law.

In determining whether a nonlawyer's activity should be considered the unauthorized practice of law, all of the following will be considered, EXCEPT whether the activity _______ A Involves legal knowledge and skill beyond that which the average layperson possesses B Is in exchange for payment or is pro bono C Is one traditionally performed by lawyers D Constitutes advice or services concerning binding legal rights or remedies

B Is in exchange for payment or is pro bono A nonlawyer may not engage in the unauthorized practice of law. Whether such activity is performed for payment or pro bono is irrelevant; a nonlawyer engaging in the practice of law will be subject to several sanctions, including injunction, contempt, and criminal conviction. Note that a lawyer may assist a person who wishes to appear pro se, or on their own behalf, in a legal matter, but this does not mean that nonlawyers may properly assist others in the practice of law, even if done voluntarily, without payment. Important considerations in determining whether the activity constitutes the practice of law include whether the activity: (i) involves legal knowledge and skill beyond that which the average layperson possesses; (ii) constitutes advice or services concerning binding legal rights or remedies; and (iii) is one traditionally performed by lawyers.

Which of the following statements regarding fee splitting is most correct? A Lawyers may split fees only with current partners and associates within a law firm B Lawyers generally should not split a legal fee with other lawyers, subject to a few exceptions C Lawyers are generally free to enter into fair fee-splitting arrangements with other lawyers at their discretion D Lawyers are absolutely forbidden from splitting fees with other lawyers

B Lawyers generally should not split a legal fee with other lawyers, subject to a few exceptions As a general rule, a lawyer should not split a legal fee with another lawyer, but this general rule is subject to three exceptions discussed below. First, current partners and associates within a law firm may split legal fees, but this is not the only instance where fee splitting is permitted. Second, a law firm may also make payments to a former partner or associate under a separation or retirement agreement. Finally, two or more lawyers from different firms can split a fee if: (i) the total fee is reasonable; (ii) the split is in proportion to the services performed by each lawyer, or some different proportion if each lawyer assumes joint responsibility for the matter; and (iii) the client agrees to the split in a writing that discloses the share each lawyer will receive. If the fee splitting agreement does not fall into one of the above mentioned exceptions, then lawyers are not free to enter into a fee-splitting arrangements; it is not a matter left to the lawyers' discretion. It is incorrect to state that lawyers are forbidden from splitting fees with other lawyers.

A lawyer __________ withdraw from representing a client if the representation will require the lawyer to violate a law or disciplinary rule; a lawyer __________ withdraw from a case if a client's criminal or fraudulent conduct involves some assistance by the lawyer. A Must; may B Must; must C Must; may D May; may

B Must; must If the representation will require the attorney to violate a law or a disciplinary rule, the attorney must withdraw. Similarly, if the client's criminal or fraudulent conduct involves some assistance by the lawyer, then the lawyer must withdraw. A lawyer may withdraw from representing a client if the client persists in a course of action that involves the lawyer's services and that the lawyer reasonably believes is criminal or fraudulent. An attorney may also choose to withdraw from representing a client if the client has used the attorney's services to commit a past crime or fraud.

A complaint against a lawyer can be brought by an aggrieved client or anyone with knowledge of the misconduct. Under what circumstances may a lawyer sue the complainant for defamation on the basis of the complaint? A Only if the complaint does not ultimately lead to discipline B Never; any such complaint is considered privileged and thus cannot be the basis of an action by the lawyer against the complainant C Only if the complaint was brought by an aggrieved client, rather than an uninvolved third party

B Never; any such complaint is considered privileged and thus cannot be the basis of an action by the lawyer against the complainant Disciplinary proceedings against a lawyer begin when a complaint is made to the state disciplinary authority (usually the state bar). Complaints are often brought by aggrieved clients, but may also be brought by anyone with knowledge of the misconduct. Filing a complaint against a lawyer is considered privileged. Thus, it cannot be the basis of a defamation action by the lawyer against the complainant. The privilege applies whether the complaint is brought by a client or another person with knowledge of the misconduct. This is true even if the complaint does not ultimately lead to discipline.

The State A Bar has established an Interest on Lawyers' Trust Accounts ("IOLTA") program, whereby lawyers deposit client trust funds into special client trust accounts that pay interest to the State A Bar, which then uses the money to help fund legal services for poor people. The program requires lawyers to deposit a particular client's funds in an IOLTA account unless the funds would earn more than $50 in interest during the time they are entrusted to the lawyer. If the client's funds would earn more than $50 in interest during that time, the lawyer must deposit them in a separate interest-bearing trust account and pay the interest to the client. A lawyer settled a personal injury case brought by her client. The defendant sent the lawyer a check for $9,000. Because she was leaving that day for a one-month vacation, the lawyer instructed her assistant to deposit the check in the lawyer's IOLTA account. The assistant is authorized to make deposits to and withdrawals from the account. The lawyer did not tell her assistant to notify the client that the check had arrived. When the lawyer returned a month later, she notified the client that the check had been received, and the client came to the lawyer's office that same day to collect the $9,000. At the prevailing rate of interest, the $9,000 would have earned $40 during the month that the lawyer was gone. Was the lawyer's handling of the matter proper? A No, because she should have instructed her assistant to deposit the check in a separate trust account that would earn interest for the client. B No, because she should have instructed her assistant to notify the client promptly that the check had arrived. C Yes, because she handled the matter in accordance with the State A IOLTA program. D Yes, because the client was not harmed.

B No, because she should have instructed her assistant to notify the client promptly that the check had arrived. The lawyer's handling of the matter was not proper because she should have made sure that the client was promptly notified that the check had arrived. When someone delivers money to a lawyer to hold for the lawyer's client, the lawyer must promptly notify the client that the money has arrived. [ABA Model Rule 1.15(d)] Had the client known that the money had arrived, she could have promptly collected it and put it to her own use. (A) is wrong because the IOLTA program required the funds to be deposited in an IOLTA account. (C) is wrong because although the lawyer did comply with the IOLTA requirement, she failed to take appropriate steps to have the client promptly notified that the money had arrived. (D) is wrong because the client was harmed; she was deprived of the use of the money during the month that the lawyer was on vacation.

A personal injury lawyer and an orthopedic surgeon are good friends, and they have a high mutual regard for each other's professional abilities. One day on the golf course, they made a reciprocal referral agreement: whenever the lawyer has a personal injury client with need for an orthopedic surgeon, the lawyer promised to refer the client to the surgeon. Similarly, whenever the surgeon has an injured patient with a need for a personal injury lawyer, the surgeon promised to refer the patient to the lawyer. The agreement was oral, not written, and there was no mention of an expiration date; both women simply assumed that the agreement would continue indefinitely until one or the other wanted to end it. Likewise, they did not discuss whether the agreement would be exclusive; both women simply assumed that neither of them would refer someone to a competitor of the other. Was it proper for the lawyer to make this agreement with the surgeon? A No, because the agreement was not reduced to writing. B No, because the agreement was of an indefinite duration. C No, because a lawyer must not give anything of value to a person for recommending her services. D No, because a lawyer must not enter into a reciprocal referral agreement with a nonlawyer.

B No, because the agreement was of an indefinite duration. ABA Model Rule 7.2(b)(4) permits a lawyer to make a reciprocal referral agreement with another lawyer, or with a nonlawyer professional, if the agreement is not exclusive and the referred person is told about the agreement. However, comment 8 to ABA Model Rule 7.2 cautions lawyers that such an agreement should not be indefinite in duration. (A) is wrong because ABA Model Rule 7.2(b)(4) does not require a reciprocal referral agreement to be in writing. (C) is wrong because reciprocal referral agreements are one of four exceptions to the general rule that a lawyer must not give something of value for a referral. [See ABA Model Rule 7.2(b)] (D) is wrong because ABA Model Rule 7.2(b)(4) expressly permits reciprocal referral agreements with nonlawyer professionals.

A 12-year-old boy was badly injured when he was struck by a dump truck owned by a construction company and driven by the company's employee. The boy and his parents sued the construction company and the employee. The first count of their complaint alleges that the employee drove negligently while acting within the scope of his duties for the construction company, and that the construction company is therefore liable for the boy's injuries. The second count alleges that the employee drove negligently while on a frolic of his own, and that the employee is therefore liable for the boy's injuries. The construction company hired a lawyer to defend both the construction company and its employee. The lawyer conducted a careful investigation of the facts and concluded that the employee was in no way negligent; he was driving slowly and carefully when the boy suddenly ran out into traffic from between two parked cars. The lawyer further concluded that the employee was acting within the scope of his duties when the accident happened. The lawyer concluded that he could win the case because of the lack of negligence, and that he could effectively represent both the employee and the construction company. He then carefully explained the potential conflicts of interest to both of them and obtained their informed consent, confirmed in writing, to the joint representation. After exhaustive discovery proceedings, the lawyer remained convinced that the employee was not negligent, but he nonetheless explained the potential conflicts to the employee and the construction company a second time and again obtained their informed consent, confirmed in writing, to the joint representation. Three weeks before the case was scheduled for trial, counsel for the plaintiffs moved to disqualify the lawyer due to a conflict of interest between the employee and the construction company. Must the trial judge disqualify the lawyer? A No, because there is no actual or potential conflict between the employee and the construction company. B No, because the employee and the construction company gave informed consent, confirmed in writing, to the joint representation. C Yes, because the potential conflict creates an appearance of impropriety. D Yes, even though the employee and the construction company gave informed consent, confirmed in writing, to the joint representation.

B No, because the employee and the construction company gave informed consent, confirmed in writing, to the joint representation. The judge must not disqualify the lawyer because the employee and the construction company gave informed consent, confirmed in writing, to the joint representation. The interests of the employee and the construction company are in potential conflict with respect to the agency issue: If the employee were on a frolic of his own, the construction company will not be liable, but if he were acting within the scope of his duties, the construction company will be liable. A lawyer may represent two clients in civil litigation if their interests are potentially in conflict, provided that the lawyer: (i) reasonably believes he can represent both clients effectively; (ii) discloses the potential conflict and explains how it can harm each client; and (iii) obtains informed consent, confirmed in writing, from each client. Because it appears that the employee was not negligent, the conflict between the employee and the construction company is only potential. The lawyer apparently believes that he can represent both parties effectively and took all of the right steps to deal with this potential conflict. Because the employee and the construction company gave informed consent in writing, the trial judge should refuse to disqualify the lawyer. [ABA Model Rule 1.7(b); Restatement �122] The judge should also consider that it is the plaintiffs who seek the disqualification; one may reasonably conclude that they were trying to harass the defendants rather than serve the interests of justice. (A) is wrong because, as discussed above, there is a potential conflict between the employee and the construction company. (C) is wrong because the potential conflict has been handled properly and does not create an appearance of impropriety. Furthermore, avoiding the "appearance of impropriety" is an outdated concept from the old ABA Model Code. The term does not appear in the ABA Model Rules because its meaning is too uncertain to be useful in a professional code of conduct. (D) is wrong because informed consent, confirmed in writing, is sufficient in this situation. A client cannot be asked to consent when a disinterested lawyer would conclude that the client's interests would not be adequately protected in light of the conflict. Here, the conflict is merely potential; thus, a disinterested lawyer could conclude that the clients can agree to the representation. Consequently, the employee's and the construction company's informed consent, confirmed in writing, are sufficient to solve the potential conflict of interest.

A client lives in State A and is a regular client of an attorney who is admitted to practice only in State A. When the client was on vacation in distant State B, she was injured in a car accident caused by a resident of State B. The client hired the attorney to represent her in a civil action against the State B driver. For reasons of jurisdiction and venue, the case had to be filed and tried in State B. The written fee agreement between the client and the attorney provided that: (1) The attorney would assume full responsibility for the case as lead lawyer; (2) The client would pay the attorney 40% of the net recovery after deduction of litigation expenses; (3) The attorney would associate a State B lawyer to serve as trial counsel in State B; (4) The State B attorney would assume responsibility only for his work as trial counsel; and (5) The attorney would pay the State B attorney an appropriate portion of the 40% contingent fee. Would it be proper for the attorney to split his fee with the State B attorney under the circumstances described above? A No, because the attorney is not admitted in State B. B No, because the share that each lawyer will receive was not disclosed in the written fee agreement. C Yes, because the State B attorney was assuming responsibility for his work as trial counsel. D Yes, because there was a written fee agreement.

B No, because the share that each lawyer will receive was not disclosed in the written fee agreement. It would not be proper for the attorney to split his fee with the State B attorney because the written fee agreement with the client does not comply with the ABA Model Rules. ABA Model Rule 1.5(e) allows a lawyer to split a fee with a lawyer who is not in his firm if: (i) the total fee is reasonable; (ii) the split is in proportion to the services rendered by each lawyer, or in some other proportion if each lawyer assumes joint responsibility for the matter; and (iii) the client agrees to the split in a writing that discloses the share that each lawyer will receive. Here, the written fee agreement did not specify the share that each lawyer will receive; thus, a fee split between the attorney and the State B attorney would be improper. (A) is wrong because there is no requirement that a lawyer be licensed in the same state as the attorney with whom he is splitting a fee. (C) is wrong because the written fee agreement with the client did not indicate the share that each lawyer will receive, and thus the agreement was improper regardless of whether the State B attorney was assuming responsibility for his work. (D) is wrong because even though there was a fee agreement, it did not comply with the ABA Model Rules.

For many years a lawyer has done business transactions work for a wealthy client. The client was recently injured in an automobile crash, and she has asked the lawyer to represent her as plaintiff in an action against the driver who injured her. The lawyer has taken some business cases to trial, but he has never handled a personal injury case. The lawyer would like to help his client and also generate some income. Which of the following would be an improper way for him to do so? A Take the case and, with the client's consent, associate a co-counsel who is competent in the field of personal injury law. B Refer the client to a competent personal injury lawyer and charge that lawyer a $1,000 forwarding fee. C Refer the client to a competent personal injury lawyer and charge the client a reasonable sum for the time spent in making the referral. D Take the case and, with the client's consent, undertake additional research to bring himself up to speed in the field of personal injury law.

B Refer the client to a competent personal injury lawyer and charge that lawyer a $1,000 forwarding fee. A "forwarding fee" is another term for a "referral fee," and payments for referrals are prohibited. [ABA Model Rule 7.2(b)] (A) is proper because a lawyer may take on a case that he is not competent to handle if he obtains his client's consent to associate a lawyer who is competent to handle it. [Comment 2 to ABA Model Rule 1.1] (C) is proper because a lawyer may refer her client to another lawyer who is competent to handle the case. Making a sound referral can take a significant amount of time, especially if the referring lawyer needs to research the backgrounds of several lawyers with whom she is not personally familiar. It is appropriate for the referring lawyer to charge her client for the time spent making the referral, subject of course to the general rule on reasonableness. [ABA Model Rule 1.5] (As a practical matter, however, many lawyers would not charge a regular client for making such a referral.) (D) is proper because a lawyer may take on a case that he is not competent to handle if he undertakes the study necessary to provide competent representation. [Comment 2 to ABA Model Rule 1.1]

In which of the following scenarios would it be mandatory for an attorney to withdraw from a case? A The attorney reasonably believes that the client's chosen course of action is criminal or fraudulent B The attorney's physical condition materially impairs his ability to continue representing the client C The client has made the attorney's work unreasonably difficult by refusing to cooperate with the attorney D The client insists on taking action that the attorney considers to be repugnant

B The attorney's physical condition materially impairs his ability to continue representing the client An attorney must withdraw if the attorney's mental or physical condition materially impairs the attorney's ability to continue representing the client. Under this scenario, withdrawal is mandatory. An attorney may also choose to withdraw from representing a client for a variety of other reasons, but these are optional withdrawals, not mandatory. An attorney may withdraw from representing a client if the client persists in a course of action that involves the attorney's services and that the attorney reasonably believes is criminal or fraudulent. Note that the lawyer is required to withdraw only if the client's criminal or fraudulent conduct involves some actual assistance by the lawyer. An attorney may withdraw from representing a client if the client insists on taking action that the attorney considers repugnant or with which the lawyer has a fundamental disagreement. Finally, an attorney may also choose to withdraw from representing a client if the client has made the attorney's work unreasonably difficult by refusing to cooperate with the attorney.

Which of the following would NOT be deemed good cause for a lawyer to seek to avoid representing a court-appointed client? A The lawyer finds the client so personally repugnant that the lawyer could not represent the client effectively B The client has been charged with a crime that makes the client extremely unpopular in the community C The lawyer has prior confidential information relevant to the client's case D The client's inability to pay would impose an unreasonable financial burden on the lawyer

B The client has been charged with a crime that makes the client extremely unpopular in the community The fact that a court-appointed client has been charged with a crime that makes the client extremely unpopular in the community is not good cause for a lawyer to seek to avoid the appointment. Lawyers have an ethical obligation to help make legal service available to all who need it. A lawyer can fulfill this obligation by accepting a fair share of unpopular matters or indigent or unpopular clients. A lawyer may seek to be excused from an appointment if to accept it would impose an unreasonable financial burden on the lawyer. A lawyer may seek to be excused from a court appointment if the lawyer finds the client or the cause so repugnant that the lawyer-client relationship would be impaired or the lawyer could not represent the client effectively. A lawyer must decline a court appointment if to accept it would require the lawyer to violate a law or disciplinary rule, such as when the lawyer has prior confidential information relevant to the client's case.

The attorney-client privilege exists for the benefit of __________. A Either the client or the attorney, depending on the nature of the privileged material B The client only C Both the client and the attorney D The attorney only

B The client only The attorney-client privilege exists for the benefit of the client only. The client is the "holder" of the privilege and the client is the one who can claim or waive the privilege. The attorney-client privilege is not for the benefit of the attorney, regardless of the nature of the privileged material. Note that the attorney has a duty to invoke the privilege on the client's behalf unless the client has waived the privilege.

Which of these is NOT a factor to consider in determining the reasonableness of an attorney's fee? A The reputation of the lawyer performing the services B The client's willingness to agree to the fee C The results obtained for the client D The nature of the relationship between the lawyer and client

B The client's willingness to agree to the fee The client's willingness to agree to the fee is not indicative of the fee's reasonableness. In theory, an attorney and client bargain at arm's length over the fee, but in practice many clients are inexperienced with attorneys' fees. Thus, in fee disputes, courts strain to give the benefit of the doubt to the client and will make an independent determination as to whether a fee is reasonable, considering the factors listed below. The factors considered in determining the reasonableness of a fee are: (i) The time and labor required; (ii) The novelty and difficulty of the questions involved; (iii) The skill needed to perform the legal services properly; (iv) The likelihood, if apparent to the client, that the work for this client will preclude the lawyer from doing fee-paying work for others; (v) The fee customarily charged in the locality for similar legal work; (vi) The amount at stake and the results obtained for the client; (vii) The time limitations imposed by the client or the circumstances; (viii) The nature and length of the relationship between the lawyer and the client; (ix) The experience, reputation, and ability of the lawyer performing the services; and (x) Whether the fee is fixed or contingent (a contingent fee can be higher because it requires the lawyer to take a gamble).

A lawyer is NOT permitted to report another's lawyer's violation of the Rules of Professional Conduct if: A The violation involves the other lawyer's personal affairs, rather than his professional conduct B The lawyer learned about the violation through a privileged communication with one of his clients C The lawyer only suspects, but has no actual knowledge of, the violation D The violation does not raise a substantial question as to the other lawyer's honesty, trustworthiness, or fitness as a lawyer

B The lawyer learned about the violation through a privileged communication with one of his clients If a lawyer learns about another lawyer's violation of the Rules of Professional Conduct through a privileged communication with the other lawyer or one of his clients, the lawyer is not permitted to report the misconduct. In fact, the lawyer would be subject to discipline for violating the confidentiality rules if he did report it. When a lawyer knows that another lawyer has violated the Rules of Professional Conduct in such a way that it raises a substantial question as to that lawyer's honesty, trustworthiness, or fitness as a lawyer, he must report the violation to the appropriate professional authority. This is true, even if the conduct involves the other lawyer's personal affairs. A lawyer may also choose to report suspected misconduct, even without actual knowledge, but he is not required to do so. A lawyer need not report other violations that do not raise a substantial question as to the other lawyer's honesty, trustworthiness, or fitness as a lawyer, but is permitted to do so.

A lawyer who limits his practice to bankruptcy law has signed up on the local court roster of attorneys who are willing to take court-appointed criminal defense matters on a pro bono basis. He has taken approximately one such pro bono criminal case each of the past 10 years, but he has won only two of them. The day after tomorrow, the lawyer will start the jury trial of a criminal defendant charged with indecent exposure. This morning, the prosecutor held a press conference, at which he told reporters that this defendant had been accused of various sex offenses on six prior occasions. The prosecutor's statements are correct, but none of the prior incidents will be admissible in evidence at the upcoming trial. The defendant's lawyer thinks that the prosecutor was simply trying to poison the jury pool by degrading the defendant. The lawyer is planning to call his own press conference at which he will give the reporters the rest of the story. The lawyer intends to explain that on all six prior occasions, the defendant was arrested but never charged, and all six arrests were made by the same police officer, who holds a personal grudge against the defendant. Which of the following is correct? A The lawyer is subject to discipline for accepting this court appointment in light of his apparent lack of talent for criminal trial work. B The lawyer's proposed statements at the press conference are proper in light of the prosecutor's prior statements to the press. C The lawyer is subject to discipline for accepting court appointments in criminal matters when his active practice is limited to bankruptcy law. D The lawyer will be subject to discipline if he holds the press conference and makes the statements described above.

B The lawyer's proposed statements at the press conference are proper in light of the prosecutor's prior statements to the press. A lawyer is permitted to "make a statement that a reasonable lawyer would believe is required to protect a client from the substantial undue prejudicial effect of recent publicity not initiated by the lawyer or the lawyer's client." [ABA Model Rule 3.6(c)] The lawyer is permitted to make these clarifying statements in response to the true, but incomplete, statements made by the prosecutor at a press conference. Thus, (D) is incorrect. (A) and (C) are incorrect because the lawyer has 10 years of experience handling pro bono criminal defense cases, and his win-loss ratio may be reflecting the underlying merits of the cases rather than the lawyer's trial ability.

Which of the following rights does NOT apply to a lawyer accused of professional misconduct? A The right to counsel B The right to exclude evidence obtained from an illegal search C The right to proper notice D The right to cross-examine adverse witnesses

B The right to exclude evidence obtained from an illegal search The exclusionary rules of criminal law do not apply to disciplinary proceedings. Thus, a lawyer accused of professional misconduct does not have the right to exclude evidence obtained from an illegal search. Such evidence is admissible in a disciplinary proceeding. An accused lawyer is entitled to procedural due process, which means that she has the right to counsel, to proper notice, to be heard and introduce evidence, and to cross-examine adverse witnesses. In addition, the hearing must be limited to the charges made in the complaint.

When must a lawyer reject a potential client? A When the client cannot currently provide enough facts to support his case B When the client's motive is harassment C When the client's case clearly is unsupported by current law D When the client's cause is unpopular

B When the client's motive is harassment A lawyer is subject to discipline for bringing an action, conducting a defense, asserting a position, or taking other steps if the client's motive is to harass or maliciously injure any person. Thus, a lawyer must reject any case where he believes that harassment is the prospective client's motive. A lawyer who is serving as an advocate in a legal proceeding must not take a position that is either factually or legally frivolous. A position is not frivolous if the lawyer can make a good faith argument that the facts are as claimed or that the present law should be changed. Thus even if a client's case clearly is unsupported by current law, a lawyer may bring an action arguing for a change of law. Furthermore, a position is not frivolous merely because the client cannot currently provide enough facts to support a case, as the lawyer can attempt to develop the facts during discovery. Lawyers have an ethical obligation to help make legal service available to all who need it and thus should not reject a client simply because the client's cause is unpopular.

When may a lawyer properly choose to delay transmission of pertinent information to her client? A When necessary to best serve the interests of a third party B When the lawyer believes that the client would be likely to react imprudently to the information C Never, because a lawyer is under a strict duty to transmit all pertinent information to her client immediately D When transmitting the information would be inconvenient to the lawyer

B When the lawyer believes that the client would be likely to react imprudently to the information A lawyer may delay the transmission of information to a client if the client would be likely to react imprudently to an immediate communication. For example, if a lawyer fears a client may become suicidal upon hearing the information, the lawyer may choose to keep the information from the client until the client is in a different mental state. Thus, it is incorrect to state that the lawyer may never withhold information from a client. As discussed above, in some instances, it may be necessary for the lawyer to do so. In addition, a court rule or order may also forbid a lawyer from sharing certain information with a client, and the lawyer must comply with such a rule or order. The lawyer must not, however, withhold information from a client solely to serve the lawyer's or a third person's interest or convenience.

A lawyer who fails to report another lawyer's misconduct that raises a substantial question as to that lawyer's honesty, trustworthiness, or fitness as a lawyer __________. A Will suffer no repercussions because self-policing in the legal profession is voluntary B Will herself be subject to discipline because she has violated the rule requiring disclosure C Will suffer no repercussions because she is upholding the tradition of privilege between attorneys D Will herself be subject to discipline for failing to intervene in an attempt to stop the misconduct from occurring

B Will herself be subject to discipline because she has violated the rule requiring disclosure A lawyer who knows that another lawyer has violated the Rules of Professional Conduct in such a way that it raises a substantial question as to that lawyer's honesty, trustworthiness, or fitness as a lawyer must report the violation to the appropriate professional authority. A lawyer who fails to report this type of misconduct is herself subject to discipline for violating the rule requiring disclosure. There is no requirement that a lawyer intervene in an attempt to stop the misconduct from occurring. She need only report the conduct to the proper authority. The legal profession prides itself on being self-policing. One element of a self-policing group is that each member of the group is obligated to report misconduct by the other members. There is no tradition of privilege between attorneys. Privilege would only apply if there were an attorney-client relationship between the two or if the lawyer learned about the violation through a privileged communication with the other lawyer or one of his clients.

A client hired an attorney to put together a complex real estate syndicate. In connection with that work, the client disclosed to the attorney a great deal of confidential information about the client's financial affairs. When the task was about half completed, the attorney's wife was killed in a car accident and his family's house burned down, all in the same week. The attorney was so emotionally and physically drained that he felt he could not competently continue with the work for his client. The client refused to allow the attorney to withdraw. The attorney begged the client to allow him to turn the files over to his law partner, an excellent real estate lawyer who was completely trustworthy and perfectly competent to handle the matter. The client refused to allow his files to be turned over to any other lawyer and insisted that the attorney himself promptly complete the work. What should the attorney do? A Turn the files over to his partner, and remain available to assist his partner to the extent possible. B Withdraw and turn the client's files over to the client. C Set the client's work aside until he recovers from the ills that have befallen him. D Continue with the matter and do the best that he can under the circumstances.

B Withdraw and turn the client's files over to the client. A lawyer must withdraw if the lawyer's physical or mental condition will materially impair his ability to represent the client. [ABA Model Rule 1.16(a)(2)] The client may be right in thinking that hard work will be good for the attorney, but the attorney has to be the ultimate judge of his own physical and mental capacity to carry on. If the attorney believes that his condition prevents him from serving the client competently, he must withdraw regardless of what the client wants. (A) is wrong because the files include confidential information about the client's financial affairs, and the attorney cannot turn them over to his law partner against the client's express wishes. [ABA Model Rule 1.6] (C) is wrong because the client has asked the attorney to complete the work promptly. The attorney's recovery may take months or years. The attorney must not continue representing the client unless he can complete the work with reasonable diligence and promptness. [ABA Model Rule 1.3] (D) is wrong because, as discussed above, if the attorney believes his mental and physical conditions prevent him from serving the client competently, he must withdraw.

A lawyer is defending a marine supply company in a civil action brought by the state attorney general under a statute that makes it a civil offense for any person or business entity to bribe or give a kickback to a state official. The statute authorizes fines of up to $100,000 per transaction for any violation. The marine supply company has a strict corporate policy that prohibits its employees from bribing or giving kickbacks to anyone. Employees who violate the policy are subject to immediate discharge and are required to indemnify the marine supply company for any loss it suffers as a consequence of the violation. The attorney general has noticed the depositions of dozens of the marine supply company's employees. One of these employees, prior to his recent retirement, was the sales manager of the marine supply company. The lawyer met with this employee to prepare him for his deposition. At the outset of the interview, the lawyer agreed to represent the employee without charge, and the lawyer told the employee that anything said between them would be confidential. During the interview, the lawyer asked the employee whether he had ever bribed any state officials. The employee confessed that he had, but said it had been necessary because all of the company's competitors were doing it, too. What course of action may the lawyer pursue at this point? A Withdraw from the case and inform the attorney general what the employee said. B Withdraw from the case and keep the employee's statement in confidence. C Withdraw from representing the employee and inform the marine supply company what the employee said. D Continue in the case, inform the marine supply company what the employee said, and advise the marine supply company to seek prompt settlement.

B Withdraw from the case and keep the employee's statement in confidence. The lawyer may withdraw from the case and keep the employee's statement in confidence. When an organization is the lawyer's client, the lawyer owes a duty of loyalty to the organization. When the interests of the organization and its constituents conflict, the lawyer should remind the person that the lawyer represents the organization and not the person. It would be appropriate for the lawyer to remind the person that communications between them may not be protected by the attorney-client privilege, and that the person may want to obtain independent counsel. [ABA Model Rule 1.13] Here, the lawyer should not have asked the question unless he was prepared for an affirmative answer. He should have known before asking that if the answer was yes, the employee's interests and the marine supply company's interests would conflict; thus, the lawyer should not have offered to represent the employee, and certainly should not have promised to keep the employee's statements in confidence. (Note that the lawyer could be subject to discipline for this conduct.) The issue here, however, is what course of action the lawyer may now take. Now that the lawyer has agreed to represent the employee, and the employee has confessed in confidence, the only thing the lawyer can do is withdraw from the matter entirely and keep the employee's confession in confidence. [See ABA Model Rule 1.9] (A) is wrong because he would violate the employee's confidence by disclosing the confession to the Attorney General. (C) and (D) are wrong because revealing the employee's confession to the marine supply company would also violate the employee's confidence.

Does a true retainer fee differ from a payment in advance? A No, both terms are used interchangeably to indicate payment for services yet to be rendered B Yes, a true retainer fee is money paid solely to ensure the availability of a lawyer, whereas an advance is a payment for services yet to be rendered C No, both terms are used interchangeably to indicate money paid solely to ensure the availability of a lawyer D Yes, a true retainer fee is money paid for services yet to be rendered, whereas an advance is solely to ensure the availability of a lawyer

B Yes, a true retainer fee is money paid solely to ensure the availability of a lawyer, whereas an advance is a payment for services yet to be rendered A true retainer fee is money that is paid solely to ensure the availability of the lawyer, and the lawyer who is fired or withdraws generally need not refund the retainer fee. In contrast, when a lawyer requires advance payment for services yet to be rendered, she must refund any unearned part of the advance if she is fired or withdraws.

A judge sits on a federal appellate court. He and two other federal judges heard a diversity of citizenship case in which they were required to interpret a state statute concerning the marital communications privilege. The judge's two colleagues wrote the majority opinion, in which they concluded that the statute gives only the witness-spouse the right to claim the privilege. The judge wrote a vigorous and scholarly dissent, arguing that the statute gives both spouses the right to claim the privilege. Later, a state senator introduced a bill to amend the statute to reflect the judge's position. The state senate invited the judge to testify about the public policy reasons for giving both spouses the right to claim the privilege. May the judge testify? A Yes, but only if the two judges who wrote the majority opinion are also allowed to testify. B Yes, because a judge may engage in activities designed to improve the law. C No, because a judge must not become involved in politics, subject to certain exceptions that do not apply here. D No, because a judge is not allowed to make public statements about disputed propositions of law, except when acting in his judicial capacity.

B Yes, because a judge may engage in activities designed to improve the law. The judge may testify at a public hearing in connection with matters concerning the law. [CJC Rule 3.2(A)] (A) is wrong because there is no rule requiring "equal time." (C) is wrong because it is overbroad. The general rule against judicial involvement in politics limits only some types of political activities, not including legislative testimony. [CJC Canon 4] (D) is wrong because, with respect to issues that are likely to come before the court, a judge is prohibited from making pledges, promises, or commitments that are inconsistent with the impartial performance of his duties. [CJC Rule 4.1(A)(13)] That Rule does not apply here because the judge's testimony, which would be designed to improve the law, would not constitute a promise that is inconsistent with the performance of his adjudicative duties.

A lawyer practices real estate law in an old-fashioned jurisdiction in which almost every real estate transaction requires the services of one or more lawyers. The lawyer is also licensed by the state as a real estate broker. The lawyer conducts her law practice and her real estate brokerage business in a single office, using one secretary and one paralegal as her support staff. The lawyer specializes in small, relatively old apartment buildings that are not in peak condition. They make good investments because they can be bought cheap, fixed up, and leased at favorable rates. When the lawyer hears that an owner of a suitable building is looking to sell, she visits them in person and asks them to consider using her to find a buyer. After an owner signs her up as their real estate broker, the lawyer lets them know that she can also do the necessary legal work—the title search, the financing documents, the land transfer documents, and the like. Is the lawyer subject to discipline? A Yes, because a person who is engaged in full-time law practice must not conduct a related business from a single office. B Yes, because a person who offers legal services along with real estate brokerage services must not engage in face-to-face solicitation of persons known to need real estate brokerage services. C No, because the lawyer's real estate brokerage services are ancillary to her law practice, and the two operations are conducted from a single office. D No, so long as her face-to-face pitch to the owners of apartment buildings is truthful and not misleading.

B Yes, because a person who offers legal services along with real estate brokerage services must not engage in face-to-face solicitation of persons known to need real estate brokerage services. The lawyer's real estate brokerage business is a "law-related service" within the meaning of ABA Model Rule 5.7, and the lawyer offers her real estate brokerage services "in circumstances that are not distinct from" her provision of legal services. [See ABA Model Rule 5.7(a)(1)] That means that she must follow the rules of legal ethics in her real estate brokerage work as well as her law work. [Id.] One of the legal ethics rules forbids a lawyer from initiating live person-to-person contact with a person known to need legal services in a particular matter when a significant motive for doing so is the lawyer's pecuniary gain. [See ABA Model Rule 7.3(b)] Therefore, the lawyer must not initiate face-to-face contact with potential real estate clients to interest them in using her brokerage services. (A) is wrong because it overstates the rule expressed in ABA Model Rule 5.7. (C) is wrong because it turns ABA Model Rule 5.7 on its head—because the lawyer is offering her ancillary service in circumstances that are not distinct from her legal service, she must follow the legal ethics rule for both kinds of service. (D) is wrong because the lawyer's face-to-face pitches violate the no-solicitation rule even if her statements are truthful and not misleading. [Compare ABA Model Rule 7.1 with ABA Model Rule 7.3(b)]

A lawyer regularly represents a manufacturer of electric kitchen appliances. One morning the president of the manufacturing company called the lawyer and asked if the lawyer had seen the newspaper story about a woman who was electrocuted when she opened the door of her dishwasher. The company president stated that he believed the dishwasher was one that his company had manufactured. The company president also stated that he found some quality control records from that period which reflected that some dishwashers left the plant without proper testing. He continued that the records should have been shredded, but somehow had been overlooked, and said that he intended to send the records to the shredder immediately unless the lawyer told him that he could not. Must the lawyer advise the president to keep the records? A Yes, unless the company has a clearly established policy of shredding quality control records after two years. B Yes, because the records have potential evidentiary value if the company gets sued. C No, because at this point there is no litigation pending against the company respecting this matter. D No, unless it was certain that the company was the manufacturer of the dishwasher in question.

B Yes, because the records have potential evidentiary value if the company gets sued. The lawyer must advise the president to keep the records because the records have potential evidentiary value if the company is sued. A lawyer must not counsel or assist a person to destroy material that has "potential evidentiary value." [ABA Model Rule 3.4(a)] Although it is not certain that the company manufactured the dishwasher in question, the president said he was "pretty sure that it was one of ours." Furthermore, it is not certain that the company will be sued if it was one of their washers, but the chances are good that it will be. If commencement of proceedings can be foreseen, the documents have potential evidentiary value and cannot be destroyed. [Comment 2 to ABA Model Rule 3.4] (A) is wrong because even if the company's records retention program called for the routine shredding of these records long ago, they were not shredded then, and they have potential evidentiary value now. (C) is wrong because commencement of proceedings is foreseeable. (D) is wrong because the standard is backward. The records should be preserved until the company is certain that the dishwasher in question was not manufactured by the company.

A Hollywood movie producer was charged under a criminal statute for unfair trade practices, and now faces a civil claim under the same statute. The producer retains an attorney to represent him in both suits. The attorney is a nationally known defense attorney who has represented many famous people. Most recently, he defended a celebrity in a notorious murder case that held the country rapt for several weeks. The attorney explains to the producer that the representation is very complex and would take a majority of his time for several months. Given the attorney's steep hourly rate, the producer's legal fees would likely be around $1 million. The producer is short on cash and makes the following proposal: If the attorney will represent him in both the civil and criminal suits, the producer will produce a movie based on the attorney's most famous past cases, told from the attorney's viewpoint. The attorney would have complete creative control and would be entitled to all of the movie's profits, which could be anything from $0 to $100 million. The producer had his personal attorney draw up a proposal to this effect and submitted it to the attorney. Assuming that the attorney receives any consent necessary from his former clients who might be portrayed in the movie, is this proposed arrangement proper? A Yes, but only if the payment from the movie profits is for the civil suit only. B Yes, but only if the ultimate amount paid to the attorney is not excessive in light of the work done. C No, because any amount over $1 million is clearly excessive, and this arrangement could be worth $100 million. D No, because a lawyer must not acquire media rights to a story concerning the lawyer's representation of a client.

B Yes, but only if the ultimate amount paid to the attorney is not excessive in light of the work done. This arrangement is proper if the fee is reasonable under the circumstances. As long as the fee paid does not turn out to be excessive, taking into account the attorney's risk of not being paid, the delay in payment, etc., this arrangement is acceptable. An attorney may enter into a business relationship with a client, provided certain safeguards, such as an opportunity to consult with independent counsel, are used. [ABA Model Rule 1.8(a)] (A) is wrong because although it is improper to use a contingent fee in a criminal case, this fee is not a contingent fee. Whether the attorney gets paid does not depend on the outcome of the case; it depends on how well the movie does. (C) is wrong because it is not clear that any amount over $1 million is excessive, given that the attorney is risking that he will be paid nothing and is delaying payment by a substantial period of time. (D) is wrong because it misstates or incompletely states the Rule. The Rule is that prior to the conclusion of the representation of the client, an attorney cannot acquire media rights to a story based substantially on information relating to the representation. In this case, the movie does not relate to the current representation, rather it involves past representations in the attorney's career. While the attorney could not disclose any information related to those representations without the clients' consents, the acquisition of such media rights is not improper.

An attorney was representing the plaintiff at a bench trial of a civil action pending before a judge. Midway through the plaintiff's case-in-chief, the judge called the attorney into his chambers. The judge told the attorney that he thought the attorney's case was very weak, but that he could be mistaken because he was distracted by money troubles. The judge went on to say that if he could get a $50,000 loan, he would feel much better. The attorney responded that he would be happy to loan the judge $50,000 to help him out as a friend. Later that afternoon, a messenger delivered an envelope containing $50,000 in cash to the judge's chambers. No mention was made of a promissory note, a repayment date, or an interest rate. Two days later, the plaintiff settled his lawsuit so the judge never had to decide the case. Three months later, the judge repaid the $50,000 to the attorney, together with interest at the market rate. Is the attorney subject to criminal liability for lending the money to the judge? A Yes, if it is proven that the judge intended to induce the attorney to make the loan in return for a decision in favor of the plaintiff. B Yes, if it is proven that, in making the loan, the attorney intended to induce the judge to decide the case in favor of the plaintiff. C No, because as the matter turned out, the judge never had to decide the plaintiff's case. D No, because the judge repaid the loan with interest.

B Yes, if it is proven that, in making the loan, the attorney intended to induce the judge to decide the case in favor of the plaintiff. The attorney is subject to criminal liability if he intended to induce the judge to decide the case in the plaintiff's favor. The common law crime of bribery consists of the corrupt payment or receipt of anything of value in return for official action. The $50,000 loan was obviously a thing of value. Thus, if the attorney intended the loan as an inducement to the judge to decide the case in favor of the plaintiff, then the attorney is guilty of bribery. (A) is wrong because in deciding whether the attorney is subject to criminal liability, it is the attorney's intent that counts, not the judge's intent. (C) is wrong because the attorney's crime was complete when he gave the loan, even though the judge never had to decide the case. (D) is wrong because bribery does not require an outright gift; a $50,000 loan is a thing of value, especially a loan with no repayment date, no promissory note, and no interest specified.

May a lawyer accept property as payment for services? A No, because such an arrangement is a conflict of interest B Yes, provided that it does not involve a proprietary interest in the cause of action or subject of litigation C Yes, this is always a permissible arrangement

B Yes, provided that it does not involve a proprietary interest in the cause of action or subject of litigation A lawyer may accept property in return for services (e.g., an ownership interest in a business), provided that this does not involve a proprietary interest in the cause of action or subject of litigation contrary to ABA Model Rules. Such an arrangement is subject to scrutiny as a conflict of interest because it may be a business transaction between the lawyer and the client, but it is not always impermissible.

A client hired a lawyer to draft a will for him. The client willed his entire estate to a 43-year-old widow. The client told the lawyer in confidence that he was neither a relative nor a friend of the widow. The client explained that he felt a moral obligation to the widow because he had killed her husband, and he had never become a suspect or confessed his sin to anyone. One day after signing the will, the client committed suicide. In due course, all of the client's assets were distributed to the widow, and the probate court closed his estate and discharged his executor. The lawyer never told the widow or anyone else that the client had confessed to killing the widow's husband. Now, a few years later, an enthusiastic young prosecutor is charging an innocent man with murdering the widow's husband in the first degree with aggravating circumstances, and the prosecutor is seeking the death penalty. May the lawyer voluntarily tell the innocent man's defense counsel what his client told him in confidence about killing the widow's husband? A Yes, the lawyer not only may, but he must, tell the defense counsel what the client told him. B Yes, the lawyer may tell, but he would not be subject to discipline if he decides not to do so. C No, the lawyer would be subject to discipline if he told defense counsel because the attorney-client privilege survives the death of the client. D No, because the client's confidential confession to the lawyer would be inadmissible hearsay if offered against the prosecution in the murder trial.

B Yes, the lawyer may tell, but he would not be subject to discipline if he decides not to do so. The controlling doctrine in this case is the lawyer's ethical duty of confidentiality, not the attorney-client privilege. The lawyer needs to know whether he can voluntarily reveal the client's confession, not whether he would be forced to do so if he were put on the witness stand in a court. ABA Model Rule 1.6(b)(1) states the applicable exception to the ethical duty of confidentiality: A lawyer may reveal confidential information if the lawyer reasonably believes that doing so is necessary to prevent reasonably certain death or substantial bodily harm. One might quibble whether the innocent man's death is "reasonably certain" when his trial has not even started, but surely the ethics rule should not be read to require the innocent man to order his last meal before being loosed from the executioner's grip. (A) is wrong because ABA Model Rule 1.6(b)(1) gives the lawyer discretion to reveal the client's confession; the Rule does not force him to do so. [See comment 15 to ABA Model Rule 1.6] (A few states go farther and require disclosure to prevent death or substantial bodily harm, but they are a small minority.) (C) is wrong for two reasons. First, the applicable doctrine is the ethical duty of confidentiality, not the attorney-client privilege. Second, even if the privilege were the applicable doctrine, who could claim it in this situation? The client cannot because he is dead. The client's executor cannot because the client's estate was closed and the executor was discharged. The lawyer cannot claim it because a lawyer's right to claim the privilege is only derivative from the client. (D) is wrong for two reasons. First, the admissibility of this hearsay is irrelevant to the ethics issue. Second, the client's confession would likely be admissible if offered by the innocent man against the prosecution because it is a declaration against penal interest by an unavailable declarant, and the client's will and suicide are independent evidence of the confession's trustworthiness. [See Fed. R. Evid. 804(b)(3); see also Chambers v. Mississippi, 410 U.S. 284 (1973)—due process violation where another man's confession was excluded in a murder trial]

What is the most accurate definition of a contingent fee? A A fee paid solely to ensure the availability of a lawyer B A fee that equals a percentage of the client's actual recovery in a case C A fee that is collected only if the matter is resolved in the client's favor D A fee that is contingent on the client's ability to pay

C A fee that is collected only if the matter is resolved in the client's favor Under a contingent fee agreement, the lawyer collects a fee only if the matter is resolved in the client's favor. Often, a contingent fee is expressed as a percentage of the client's eventual recovery in the case, but it need not be; an otherwise proper contingent fee may still be proper even if there is no res, or pool of money, from which the fee can be paid. Some lawyers may choose to offer discounted fees that are contingent on the client's ability to pay, but this is not what is generally meant by the term contingent fee. A fee paid solely to ensure the availability of a lawyer is called a true retainer fee.

A state university receives 45% of its annual budget from the state. The other 55% of the budget comes from private sources. The university is chartered by the state constitution, and it is regarded for all purposes as a unit of the state government. The governing body of the university is its board of overseers, a group of 17 citizens. The chief executive officer of the university is the chancellor, and the chief legal officer is the general counsel. The university has always strived for a student body and faculty that are diverse in age, politics, wealth, race, nationality, religion, sex, and sexual orientation. One year ago, the voters passed a ballot initiative that prohibits all units of the state government, including the university, from considering a person's race when offering employment or admission to school. The initiative prohibits giving any state funds to a governmental unit that violates the initiative. With reluctance, the university board of overseers adopted a new university-wide regulation that requires all admissions officers and hiring committees to obey the initiative. The state supreme court sustained the constitutionality of the initiative, and the United States Supreme Court denied certiorari. A lawyer is one of 15 attorneys in the university general counsel's in-house law office. The general counsel assigned the lawyer to work with the university's admissions office to develop new admissions criteria that will comply with the initiative. At the outset, the lawyer reminded the admissions director that she was not his lawyer, but rather the university's lawyer. The admissions director told the lawyer that despite any new admissions criteria, he would continue to consider race because he believed that was the right thing to do. Deep in her heart, the lawyer agrees with the admissions director. Which of the following may the lawyer do in responding to this situation? A Keep the admissions director's statement in confidence, even if she reasonably believes that the university is likely to lose its state funding as a consequence. B Promptly disclose the admissions director's statement to the state attorney general, who is the official in charge of enforcing the voter initiative. C Attempt to convince the admissions director to obey the voter initiative, and if he refuses, then disclose the situation to the university's general counsel. D Anonymously leak the admissions director's statement to the university's board of overseers.

C Attempt to convince the admissions director to obey the voter initiative, and if he refuses, then disclose the situation to the university's general counsel. ABA Model Rule 1.13 governs this question. The lawyer has been informed about the stated intent of a university admissions director to violate the voter initiative in a manner that imperils 45% of the university's funding. The lawyer began her conversation with the admissions director in a proper manner by reminding him that she is the university's lawyer, not his lawyer. However, the lawyer may not allow her personal views about the use of race in university admissions to affect how she responds to the director's stated intent. Here, the voters have spoken by passing the initiative, the university's board of overseers has acquiesced in the initiative, and the constitutional challenge to the initiative has failed. If the lawyer feels strongly enough about the issue to resign her position, she may do so [see ABA Model Rule 1.16(b)(4)], but she cannot remain in the general counsel's office while subverting the voter initiative. Thus, (A) is wrong. (C) is correct; if the lawyer cannot convince the admissions director to obey the voter initiative, she must "refer the matter to higher authority in the organization." [ABA Model Rule 1.13(b) and comment 4] In this instance, the lawyer's immediate boss, the general counsel, is the obvious first choice. ABA Model Rule 1.13 requires the lawyer to report up the chain of command, not leap immediately to the top rung. [Id.] Only if her report to the general counsel proves futile should she take the matter to the board of overseers, and if she needs to report it to the board of overseers, she should do so forthrightly, not by an "anonymous leak" that the board would be likely to ignore. Therefore, (D) is wrong. (B) is wrong because it appears to allow the lawyer to report first to the state attorney general, who should be regarded as outside the structure of the university for this purpose. [See ABA Model Rule 1.13(c)] True, the university is part of the state, and the attorney general is the state's highest law enforcement official, but ABA Model Rule 1.13 seeks to have legal issues resolved at the lowest possible command level, not the highest possible level. [Compare Rule 1.13(b) with 1.13(c) and comment 6]

A new associate at a law firm was asked to help a partner advise a state university on how to comply with a federal statute that requires colleges and universities to make many changes in their facilities to accommodate students with disabilities. After graduating from law school, the associate had worked on the congressional staff of a United States senator. In that role, she personally drafted a bill that was ultimately enacted as the federal statute. In light of the associate's earlier role as the drafter of the federal statute, which of the two lawyers may work on the matter? A Neither the partner nor the associate. B The partner only, and only if the associate is properly screened off from the matter. C Both the partner and the associate. D The partner only, and only if the state university consents after full disclosure.

C Both the partner and the associate. Both the partner and the associate may work on the matter, assuming the associate complies with the applicable federal statutes and regulations concerning former government employees. Drafting a piece of legislation is not regarded as a "matter" for purposes of the legal ethics rules on former government employees. [ABA Model Rule 1.11(e); ABA Formal Op. 342 (1975)] Therefore, the associate may advise the state university. Because the associate is not disqualified, neither is her firm. Thus, the partner may also work on this project. (A) is wrong because, as discussed above, the associate's congressional work disqualifies neither the associate nor the partner. (B) is wrong because it states one of the requirements for the partner's representation had the associate been disqualified. As discussed above, the associate is not disqualified and thus need not be screened off. (D) is wrong for the reasons stated above. Moreover, it is not the state university that would need protection if this were a "matter" for the purpose of disqualification under the conflict of interest rules. In that case, the associate would have been screened off, the associate would not be apportioned any part of the fee, and written notice would be promptly given to the government agency.

Commission of which of the following crimes by an attorney would most likely lead to professional discipline? A Solicitation of prostitution B Possession of marijuana where it is illegal C Intentional failure to file a personal tax return D A single incident of drunk driving

C Intentional failure to file a personal tax return A lawyer is subject to discipline for committing a criminal act that reflects adversely on his honesty, trustworthiness, or fitness as a lawyer in other respects. To constitute professional misconduct, the crime must involve some characteristic that is relevant to the practice of law. For example, crimes involving dishonesty, breach of trust, or substantial interference with the administration of justice, such as intentionally failing to file a personal tax return, and most crimes involving violence, reflect on the lawyer's fitness to practice law. The other crimes listed (solicitation of prostitution, a single offense of drunk driving, possession of marijuana), while punishable by law, do not necessarily trigger professional discipline.

A lawyer practices environmental law. He also happens to be one of the nation's leading experts on the environmental effects of filling wetlands. The state legislature has scheduled hearings on a bill to prohibit the filling of wetlands surrounding a bay. One of the lawyer's regular clients is a development company, which owns development rights to some of the wetlands in question. The development company wants to fill its wetlands so that it can build low-cost housing for underprivileged families. The development company hired the lawyer to appear as a witness at the legislative hearings and to testify in opposition to the ban on wetland filling. The lawyer appeared as a witness, identified himself as an expert on wetlands, and testified vigorously against the proposed legislation. Was the lawyer's conduct proper? A Yes, unless his testimony was contrary to his own beliefs about the environmental effects of filling wetlands. B Yes, because he is a leading expert on the environmental effects of filling wetlands. C No, unless he informed the legislators that he was appearing in a representative capacity. D No, because a lawyer must not be a witness for his client on a contested matter.

C No, unless he informed the legislators that he was appearing in a representative capacity. The lawyer's conduct was not proper unless he informed the legislators that he was appearing in a representative capacity. When a lawyer appears before a nonadjudicative body on behalf of a client, he must disclose that he is acting in a representative capacity. [ABA Model Rule 3.9] One important purpose of the Rule is to enable the members of the nonadjudicative body to assess the biases that may influence the lawyer's testimony. In this case, the legislators might well think that the lawyer was speaking in his capacity as a wetlands expert, rather than as a developer's spokesman. (A) is wrong because the lawyer should have told the legislators that he was acting in a representative capacity, whether or not his testimony was consistent with his own views. (B) is wrong; indeed, the fact that he is an expert in his own right makes his appearance particularly misleading. (D) is wrong because it invokes a nonexistent rule. There are limits on when a client's trial counsel may testify in a court proceeding, but no such limits apply in nonadjudicatory proceedings.

Under the ABA Model Rules of Professional Conduct, a lawyer is subject to discipline for committing any criminal act __________. A That reflects poorly on the legal profession B That would constitute a felony within the state where she is admitted to practice law C That reflects adversely on her honesty, trustworthiness, or fitness as a lawyer in other respects D Whatsoever, because she has taken an oath to uphold the state and federal Constitutions

C That reflects adversely on her honesty, trustworthiness, or fitness as a lawyer in other respects ABA Model Rule 8.4(b) states that a lawyer is subject to discipline for committing a criminal act that reflects adversely on her honesty, trustworthiness, or fitness as a lawyer in other respects. The other answers are incorrect because to constitute professional misconduct, the crime must involve some characteristic that is relevant to the practice of law. Crimes involving dishonesty, breach of trust, substantial interference with the administration of justice, and most crimes involving violence reflect on the lawyer's fitness to practice law. Thus it is not simply any criminal act whatsoever or any criminal act constituting a felony that will trigger professional discipline. Similarly, disciplining a lawyer based on any criminal act that reflects poorly on the legal profession arguably could include any criminal act.

An attorney in solo practice published a brochure regarding what one should do when injured. The brochure contains accurate, helpful information about obtaining proper medical treatment, recording details of the accident, notifying insurance companies, not making harmful statements, and the like. The attorney's name, address, and telephone number are printed on the brochure's cover. One afternoon, the attorney saw a pedestrian knocked down in a crosswalk by a hit-and-run driver. He and another bystander called 911 and gave the pedestrian emergency first aid until an ambulance arrived. The next day, the attorney visited the pedestrian in the hospital and gave the pedestrian a copy of his brochure. Which of the following is correct? A The attorney is subject to discipline, both for publishing the brochure and for giving the brochure to the pedestrian in the hospital. B The attorney is subject to discipline for publishing the brochure. C The attorney is subject to discipline for giving the pedestrian a copy of the brochure at the hospital. D The attorney's conduct was proper because the brochure's contents are neither false nor misleading.

C The attorney is subject to discipline for giving the pedestrian a copy of the brochure at the hospital. A potential problem arises when the content of a brochure is improper or when the printed material is used as part of an act of solicitation. Here, the content was proper, but when the attorney gave the brochure to the hospitalized pedestrian, he crossed the line into impermissible in-person solicitation. [See ABA Model Rule 7.3; see also Ohralik v. Ohio State Bar Association, 436 U.S. 447 (1978)—noting the particular potential for undue influence and overreaching in the hospital context, where individuals are often especially vulnerable] (A) and (B) are incorrect because publishing the brochure, in and of itself, is not problematic, just as the act of printing business cards is fully permissible. (D) is incorrect because the truthfulness of the brochure's contents does not counteract the attorney's impropriety in providing his contact information during a hospital visit.

What will the consequence(s) be if a court finds that an attorney's fee, agreed to at arm's length between the client and the attorney, is unreasonably high? A The court will enforce the contract and take no action against the attorney because the fee was agreed to at arm's length B The court will enforce the contract because it was agreed to at arm's length, but the attorney will be subject to discipline for violation of an ethics rule C The court will not enforce the contract for the fee and the attorney will be subject to discipline D The court will not enforce the contract for the fee, but the attorney will not be subject to discipline because the contract was agreed to at arm's length between the client and the attorney

C The court will not enforce the contract for the fee and the attorney will be subject to discipline A court will not enforce a contract for an unreasonably high attorney's fee or an unreasonably high amount for expenses, and the attorney is subject to discipline for trying to exact such a fee or amount for expenses. Even if, in theory, the attorney and client bargained at arm's length over the fee, many clients are inexperienced with attorneys' fees. Thus, in fee disputes, courts strain to give the benefit of the doubt to the client.

What duty does a lawyer have when safeguarding a client's property? A The lawyer only has an ordinary duty of care in safeguarding the property, unless the lawyer claims to be a financial specialist B The lawyer has a strict duty to safeguard all property in excess of the amount of his reasonable fee C The lawyer acts as a fiduciary and is subject to civil liability if he fails to safeguard the property D The lawyer need only act as a reasonable person would in safeguarding the property

C The lawyer acts as a fiduciary and is subject to civil liability if he fails to safeguard the property A lawyer safeguarding a client's property acts as a fiduciary and is subject to civil liability for failure to safeguard such property. When money or property belonging to a client comes into the lawyer's hands, the lawyer must not steal it, borrow it, or put it to the lawyer's own use. Furthermore, the lawyer must keep it separated from the lawyer's own money and property. A lawyer is subject to discipline for commingling the client's money or property with the lawyer's own personal or business funds or property. The lawyer's duty to safeguard a client's property in not limited to that property in excess of the amount of his reasonable fee. The lawyer must safeguard all client property. The matter of his fee is a separate issue. Since the lawyer acts as a fiduciary, his duty of care is not just an ordinary duty of care or that of a reasonable person. All lawyers act as fiduciaries, even those not claiming to be financial specialists.

A potential client went to a lawyer's office and asked him to represent her in a civil case. The lawyer agreed. The client then paid the lawyer a retainer fee for his services. A lawyer-client relationship was formed at the moment __________. A The client entered the lawyer's office with intent that the lawyer represent her B The client asked the lawyer to represent her C The lawyer agreed to represent the client D The lawyer accepted the retainer fee from the client

C The lawyer agreed to represent the client A lawyer-client relationship arises when a person manifests an intent that the lawyer provide legal services and the lawyer agrees. Thus the lawyer-client relationship here was formed at the moment the lawyer agreed to represent the client. Lawyers are generally free to refuse service to any person for any reason. Just because a prospective client enters a lawyer's office with intent that the lawyer represent her, or asks the lawyer to represent her, that is not enough to create a lawyer-client relationship. The lawyer also has to agree to provide the service. Note that a lawyer-client relationship can arise when a prospective client manifests an intent to have the lawyer represent her, and the lawyer fails to make clear that he does not want to undertake the representation, and the lawyer knows or should know that the prospective client is reasonably relying on the lawyer to provide the services. However, that is not the scenario given here. It is not necessary for payment to take place before a lawyer-client relationship is formed. Here, the lawyer agreed to represent the client and the relationship was formed at that moment, before the lawyer accepted the retainer fee from the client.

Two sisters are partners in a bakery. Their partnership agreement says that they will share the work and the profits equally. They are very close, but they constantly bicker—each claims that the other is taking an unfair share of the profits and shirking on the work. Six months ago, they hired a lawyer to act as a third-party neutral, to help them resolve their differences once and for all. At the outset, the lawyer explained that he would be strictly neutral between them; he would not be representing either one, and neither of them would be entitled to the protections afforded by an attorney-client relationship. After a long series of meetings with them (sometimes separately, sometimes jointly), the lawyer proposed a solution. The sisters liked his solution, reduced it to writing, and signed it, vowing to end their bickering forever. Six months later, the feud erupted again, worse than ever. One of the sisters asked the lawyer's law firm to represent her in a lawsuit against her partner-sister, seeking to declare the partnership at an end and to bar her partner-sister from entering the bakery premises. Which of the following is correct? A The lawyer is subject to discipline for his failed effort to serve both sisters when their interests were patently in conflict. B It would be proper for the lawyer to represent the sister in the lawsuit as she requested, even without the informed consent of her partner-sister. C The lawyer's law firm partner may represent the sister in the lawsuit as she requested, but only if her partner-sister is notified in writing, and only if the lawyer is timely screened and does not share in the fee earned in the lawsuit. D The lawyer's law firm partner would be subject to discipline for representing the sister in the lawsuit as she requested, even if the lawyer is timely screened and does not share in the fee earned in the lawsuit.

C The lawyer's law firm partner may represent the sister in the lawsuit as she requested, but only if her partner-sister is notified in writing, and only if the lawyer is timely screened and does not share in the fee earned in the lawsuit. The lawyer was not representing either sister; rather, he was acting as a third-party neutral to help them resolve their differences. ABA Model Rule 2.4 permits a lawyer to serve in that role. ABA Model Rule 1.12(c) permits screening to avoid a conflict in this situation. Therefore, (A) and (D) are incorrect. (B) is incorrect because, when a lawyer has served as a third-party neutral between two conflicted parties, he cannot later represent one of the parties in that matter, unless both parties give informed consent, confirmed in writing. [ABA Model Rule 1.12(a)]

A law professor was selected as the neutral arbitrator of a boundary line dispute between an elderly couple and the couple's next-door neighbors. The law professor decided the matter in favor of the elderly couple. Shortly thereafter, the law professor quit his teaching position and entered private law practice. The elderly couple's next-door neighbors brought suit to have the arbitration award set aside. The elderly couple asked the law professor to represent them in the suit. If the law professor takes the case, will he be subject to discipline? A No, because serving as the elderly couple's lawyer is consistent with his decision as arbitrator in their favor. B No, because by seeking to hire the law professor, the elderly couple is deemed to have consented to the conflict of interest. C Yes, because his earlier service as neutral arbitrator creates a conflict of interest. D Yes, because there is reasonable ground to doubt his impartiality in the case.

C Yes, because his earlier service as neutral arbitrator creates a conflict of interest. The law professor would be subject to discipline for representing the elderly couple because his earlier service as neutral arbitrator creates a conflict of interest. A lawyer must not represent a private client in a matter in which the lawyer has earlier participated personally and substantially while serving as an arbitrator. [ABA Model Rule 1.12(a)] (A) is wrong because the consistency of his position does not solve the conflict of interest. (B) is wrong because it does not go far enough. Informed consent, confirmed in writing, by both landowners would solve the conflict, but consent by the elderly couple alone will not suffice (and their consent was not confirmed in writing). (D) is wrong because it invokes a nonexistent rule; unlike judges, lawyers are not expected to be impartial. Here, the law professor would be acting as an advocate, not as a judge or arbitrator; thus, he should be partial to his client. The issue here is not partiality but conflict of interest.

An attorney and a licensed real estate developer, a nonlawyer, created a partnership to serve people who want to invest in commercial real estate. The real estate developer finds promising commercial real estate projects, brings together groups of investors, and works with local planning authorities to gain approval for the projects. The attorney drafts the legal documents for the projects, assists the investors with the legal technicalities, advises the investors on their tax liabilities, and does whatever legal work the investors need in connection with management and operation of the projects. The attorney and the real estate developer charge the investors a single fee for their work, and they divide the partnership profits 50%-50%. Is the attorney subject to discipline? A No, provided the investors give informed consent to the potential conflicts of interest, and such consent is confirmed in writing. B No, because the real estate developer does only development work, and the attorney does only legal work. C Yes, because the attorney and the real estate developer are partners in the business. D Yes, because she is aiding the real estate developer in the unauthorized practice of law.

C Yes, because the attorney and the real estate developer are partners in the business. The attorney is subject to discipline because she and the real estate developer are partners in the business described in the question. A lawyer is prohibited from entering into a partnership with a nonlawyer if any of the partnership activities constitutes the practice of law. [ABA Model Rule 5.4(b)] The rationale and social policy behind this Rule have been sharply questioned, but the ABA has not abandoned its traditional distrust of partnerships with nonlawyers. [See Hazard & Hodes, �45.7] (A) is wrong because although there are potential conflicts here in that the attorney appears to work partly for the investors and partly for the developer and herself in putting the real estate projects together, informed consent, confirmed in writing, by the investors will solve these conflict issues. In any event, the attorney is still subject to discipline for entering into the partnership with a nonlawyer. (B) is wrong because the division of responsibility does not solve the partnership with a nonlawyer problem. (D) is wrong because the developer is not engaging in activities that could be construed as practicing law. All of the legal work (i.e., work calling for the professional judgment of a lawyer) is done by the attorney.

The state bar association has established a peer counseling program whereby lawyers who are addicted to alcohol or other drugs can receive confidential counseling from other lawyers. The bar association's ethics rule on confidential information provides that communications between the counselor lawyer and the counseled lawyer are to be treated just like confidential communications between an attorney and client. A lawyer is addicted to alcohol and is receiving peer counseling under the program from another lawyer. The lawyer is a large, strong man, and his addiction has made him subject to periodic fits of physical violence. This afternoon, during their peer counseling session, the lawyer told his peer counselor that his client had refused to pay the fees he owes, and that he intended to punch out the client the next time he got roaring drunk. From working with the lawyer over an extended period, the peer counselor believes that he may really do it. May the peer counselor disclose the lawyer's statement to the client and the police? A No, unless the lawyer consents. B No, unless the peer counselor is certain that the lawyer will carry out his threat. C Yes, even if the lawyer objects. D Yes, because he is serving as a peer counselor, not a lawyer.

C Yes, even if the lawyer objects. The peer counselor may disclose the statement even if the lawyer objects. The state ethics rule on confidentiality treats communications between a lawyer and his peer counselor just like communications between an attorney and a client. If the counselor had heard one of her clients make this threat, she could have warned the police and the intended victim. An attorney may reveal confidential information to the extent she reasonably believes necessary to prevent reasonably certain death or substantial bodily harm. [ABA Model Rule 1.6(b)(1)] Thus, the peer counselor may warn the client and the police. (A) is wrong because the peer counselor may act to prevent the lawyer from causing substantial bodily harm. The lawyer's consent is not necessary. [ABA Model Rule 1.6(b)(1)] (B) is wrong because if an attorney reasonably believes that her client (or anyone else) is about to inflict substantial bodily harm on someone, she may take steps to prevent it, even if she is not certain that the client (or other person) will do it. (D) is wrong because the state ethics rule on confidentiality equates the peer counselor relationship with the relationship between an attorney and client; thus, the ability to disclose is the same.

An attorney is a voting member of the legislation committee of a consumer-based law reform group that drafts and advocates the passage of proposed statutes on food safety. The law reform group is currently debating a draft statute that sets quality and safety standards for growth hormones administered to chickens, turkeys, and other poultry. The attorney is also engaged in the private practice of patent law. She regularly represents a biotechnology firm. Using the techniques of genetic engineering, the biotechnology firm invents, develops, and sells a variety of patented growth hormones. The attorney herself has obtained patents on some of these hormones for the biotechnology firm. If enacted into law, the law reform group's proposed statute on poultry hormones could materially increase the biotechnology firm's hormone sales because it is the only firm whose hormones would meet the statute's quality and safety requirements. Would it be proper for the attorney, as a member of the law reform group's legislation committee, to participate in the debate on, and to cast her vote on, the proposed statute? A No, because the statute could materially benefit the biotechnology firm. B No, because the attorney may not serve as a member of the law reform group while representing the biotechnology firm. C Yes, provided that she informs the legislation committee that she represents an unnamed client whose interests could be materially benefited by the statute. D Yes, provided that she informs the legislation committee that she represents the biotechnology firm, whose interests could be materially benefited by the statute.

C Yes, provided that she informs the legislation committee that she represents an unnamed client whose interests could be materially benefited by the statute. It would be proper for the attorney to participate in the debate and cast her vote on the proposed legislation, provided that she informs the committee that she represents a client whose interests could be materially benefited by the statute. A lawyer may participate in a law reform activity that will affect the interests of the lawyer's client. [ABA Model Rule 6.4] When a lawyer knows that a client will be materially benefited by the activity, the lawyer must disclose that fact, but she need not name the client. (A) is wrong because a lawyer is not prohibited from engaging in a law reform activity that might benefit her client. (B) is wrong because a lawyer is not prohibited from participating in a law reform activity, unless the participation would create an impermissible conflict of interest. [ABA Model Rule 1.7(a)] That is not the case here. A client who hires a lawyer does not thereby purchase the right to control the lawyer's views and activities in all contexts. [See ABA Model Rule 1.2(b)] The attorney may even advocate new legislation that she thinks is sound that would harm the biotechnology firm's sales. [See ABA Model Rule 6.4] (D) is wrong because the attorney need not disclose the name of her client; simply disclosing the fact of representation will inform the legislation committee of her possible bias.

A swimming coach was charged with assault of another coach. The swimming coach hired a criminal attorney to defend him. Subsequently, the swimming coach pleaded not guilty and was released on his own recognizance. At his first trial, a jury was empanelled, and the prosecutor was almost finished presenting the testimony of her first witness when a signal from her electronic pager interrupted her. The trial judge granted her request for a short recess, at the end of which the prosecutor told the judge that her office had instructed her not to proceed with this case at this time. The judge responded that if the prosecutor stopped now, the defendant would go free. When the prosecutor indicated that she understood, the judge entered a judgment of acquittal and set the swimming coach free. Twenty days later, the prosecutor recharged the swimming coach with the same offense. The swimming coach hired his original criminal attorney to defend him. The same judge presided over the second trial. The swimming coach's attorney made no pretrial motions. This time the prosecutor did not falter, and in due course the jury at the second trial found the swimming coach guilty as charged. The judge sentenced him to prison for the period required by law, but she stayed the sentence and released him on his own recognizance pending appeal. The swimming coach reluctantly paid the criminal attorney's bill for the second trial—$5,000. However, the swimming coach hired a new lawyer for the appeal, and in due course the appellate court reversed the conviction and set aside the prison sentence. The appellate court's opinion stated it had never seen a clearer double jeopardy violation. Will the swimming coach's original criminal attorney be subject to civil liability in a legal malpractice action brought by the swimming coach for having missed the double jeopardy issue? A No, because the swimming coach never served jail time as a result of the original attorney's error. B No, even if the swimming coach proves by a preponderance of evidence that he did not commit the assault on the opposing coach. C Yes, provided that the swimming coach proves by a preponderance of evidence that he did not commit the assault on the opposing coach. D Yes, but the swimming coach can recover only nominal damages.

C Yes, provided that the swimming coach proves by a preponderance of evidence that he did not commit the assault on the opposing coach. The criminal attorney will be subject to civil liability in a legal malpractice action brought by the swimming coach for failing to object to the second trial on double jeopardy grounds. A reasonably competent criminal defense attorney would know that a defendant is put in jeopardy when a jury is empanelled and sworn, not to mention that the prosecutor started presenting her case-in-chief. The swimming coach was obviously a proper plaintiff, and he was injured by the criminal attorney's error; he should be able to recover at least part of the $5,000 attorneys' fee, plus damages for his anguish and for the reputational injury caused by the conviction at the second trial. Note that (C) provides that in the malpractice action the swimming coach must prove by a preponderance of the evidence that he was innocent of the underlying criminal offense. That is required by the law of most states that have ruled on the issue. [See Restatement �53, comment d] Observe that in this particular case, a good argument can be made for allowing the swimming coach to recover even without proof of innocence. Here, the malpractice was the criminal attorney's failure to object to the second trial. If the attorney had acted competently, the second trial would never have taken place, and the swimming coach would have lawfully gone free, even if he were unquestionably guilty of the assault. [See Levine v. Kling, 123 F.3d 580 (7th Cir. 1997)—Judge Posner's dictum] (A) and (D) are incorrect because they overlook the $5,000 fee and other less tangible injuries the swimming coach suffered. (B) is incorrect for the reasons stated above with respect to (C).

A client hired a lawyer to do the legal work in connection with a complex public securities offering. The lawyer agreed to do the work for a set hourly fee. The lawyer did a great deal of legal research, prepared numerous memoranda of fact and law, and drafted most of the documents needed for the public offering. At that point, the client became angry with the lawyer for no apparent reason and fired him. The client paid the lawyer at the agreed rate for the work the lawyer had done and demanded that the lawyer turn over to him the papers that the lawyer had prepared, including the legal and fact memoranda and the document drafts. What papers must the lawyer turn over to the client? A Only the document drafts, but not the legal and fact memoranda. B Only the legal and fact memoranda, but not the document drafts. C None of the papers, because the client fired the lawyer. D All of the papers, even though the client fired the lawyer.

D All of the papers, even though the client fired the lawyer. The lawyer must turn over all of the papers to the client. When a lawyer is fired, he must return all "papers and property to which the client is entitled." [ABA Model Rule 1.16(d)] In this case, the client is entitled to all the papers the lawyer has prepared. Under the law of many states, an attorney can assert a lien on client papers in her possession to secure the payment of her fee, but here the client has paid the lawyer for all the work the lawyer did. (A) is wrong because the lawyer must turn over the memoranda as well as the other documents. (B) is wrong because the lawyer must give the client the document drafts as well as the memoranda. (C) is wrong because the fact that the lawyer was fired, even without cause, does not in any way change the lawyer's duty to give the client all of the papers.

Which of the following would be a sufficient basis by itself to deny a candidate's application to practice law? A Membership in the Communist Party B Any record of civil disobedience C Any prior felony conviction D Any conviction of a crime involving moral turpitude

D Any conviction of a crime involving moral turpitude Conviction of a crime involving moral turpitude is sufficient by itself to cause disqualification of a candidate's application to practice law. Crimes involving moral turpitude include crimes involving intentional dishonesty for the purpose of personal gain (e.g., forgery, bribery, theft, perjury, robbery, extortion) or crimes involving violence (e.g., murder, rape, mayhem). Although many felony convictions are also crimes of moral turpitude, the mere fact that it was a felony is not controlling. The nature of the offense and the motivation of the violator are important factors in determining whether moral turpitude exists. Similarly, any record of civil disobedience is incorrect because it is too broad. For example, an arrest for nonviolent civil disobedience does not rise to the level of moral turpitude. Similarly, an applicant's mere membership in the Communist Party (when there is no showing that the applicant engaged in or advocated actions to overthrow the government by force or violence) is not sufficient to show a lack of moral character and deny the applicant admission to practice law.

Under the ABA Model Rules of Professional Conduct, which of the following is NOT a proper way for a lawyer admitted to practice in one state to temporarily provide legal services in a second state? A By requesting special permission from a local court in the second state B By representing one of her in-state clients in an arbitration proceeding in the second state C By associating herself with a local lawyer in the second state who actively participates in the matter D By paying the required annual bar dues for the second state

D By paying the required annual bar dues for the second state A lawyer who is admitted to practice law in one jurisdiction is not, without more, authorized to practice in any other jurisdiction. A lawyer is subject to discipline for practicing in a jurisdiction where she is not admitted to practice. Simply paying the required annual bar dues for the other jurisdiction is not a substitute for gaining admission to that state's bar. A lawyer admitted to practice in one state may provide legal services in a second state on a temporary basis in four situations. First, she may practice on a temporary basis in a state in which she is not admitted if she associates with a local lawyer who actively participates in the matter. Second, she may request special permission from a local court, administrative agency, or other tribunal to handle a matter in that tribunal. In a court, such permission is commonly called admission "pro hac vice," which means admission for purposes of this matter only. Third, a lawyer may mediate, arbitrate, or engage in another form of alternative dispute resolution in a state in which she is not admitted to practice if her services arise out of, or are reasonably related to, her practice in the state in which she is admitted. Finally, note that the ABA Model Rules also contain a "catch-all" category that permits a lawyer to temporarily practice out of state if the lawyer's out-of-state practice is reasonably related to the lawyer's home-state practice.

If a client asks a lawyer to do something that is either illegal or unethical, the lawyer must __________. A Refuse the request without providing any further details to the client so as to avoid any appearance of impropriety B Honor the request unless it involves a risk of harm to a third party, as it is a lawyer's duty to act as a zealous advocate on behalf of his client C Cut all ties with the client immediately and report the improper request to the proper authorities D Consult with the client and explain why he cannot do what the client wants

D Consult with the client and explain why he cannot do what the client wants If the client asks a lawyer to do something that is either illegal or unethical, the lawyer must consult with the client and explain why he cannot do what the client wants. He should not simply refuse the request without providing any further details, but should instead openly communicate with his client about the reasons why the request is improper. It is not necessary that the lawyer cut all ties with the client immediately based on such a request. Instead, the lawyer should explain to the client why the conduct is improper. Furthermore, it would likely be a violation of confidentiality to report a mere request to the authorities. Although a lawyer should act as a zealous advocate on behalf of his client, this ideal does not extend to illegal or unethical conduct.

A building contractor and his lawyer met with a landowner to negotiate a contract for construction of an office building on the landowner's property. The contractor, the lawyer, and the landowner were the only persons present at the meeting. Ultimately, the three of them worked out a written agreement, and the contractor commenced work. However, it soon became apparent that the building site required far more preparation work than the contractor had contemplated when he agreed to the contract price. The contractor and the landowner argued about who had to pay for the additional site preparation. One important issue is whether the landowner made certain oral representations to the contractor during the contract negotiating session that the lawyer attended. The contractor contends that the landowner did make the representations, while the landowner contends that he did not. The lawyer was present during the entire negotiating session, and she is virtually certain that the landowner did not make the representations. The contractor refused to proceed with construction until the landowner paid for the extra site preparation. The landowner then sued the contractor for specific performance of the construction contract. The contractor asked the lawyer to represent him as trial counsel. The lawyer should: A Agree to serve as trial counsel for the contractor because the contractor is entitled to the counsel of his choice. B Agree to serve as trial counsel for the contractor because she can refuse to testify if she is called as a witness by the landowner. C Decline to serve as trial counsel for the contractor because a lawyer is not allowed to testify in a manner that is prejudicial to her client. D Decline to serve as trial counsel for the contractor because she can foresee that she will be called as a witness.

D Decline to serve as trial counsel for the contractor because she can foresee that she will be called as a witness. The lawyer should decline to serve as trial counsel because she can foresee that she will be called as a witness. A lawyer must not act as an advocate at a trial at which the lawyer is likely to be a necessary witness. [ABA Model Rule 3.7(a)] The lawyer was the only person other than the parties present at the negotiating session. The landowner will almost certainly call the lawyer as a witness. Because the lawyer can foresee at the outset that she will likely be called as a witness, she should decline to serve as trial lawyer for the contractor, even if the contractor is willing to consent to the conflict of interest. [ABA Model Rule 1.7(a); comment 6 to ABA Model Rule 3.7] (A) is wrong because a person's choice of counsel is limited by the restraints imposed by the ethics rules. The client may choose the lawyer, but the lawyer cannot ethically accept the employment. (B) is wrong because the lawyer cannot refuse to testify if the landowner calls her to the witness stand. (C) is wrong because if a lawyer is called to the witness stand and sworn to tell the truth, she must do so, even if it is prejudicial to her client.

A police officer was charged with murder. He is alleged to have savagely beaten and ultimately killed a teenage gang member in the course of an arrest. Neither the police department nor the officer's union was willing to provide legal counsel for his defense, and the officer himself lacked funds to hire private counsel. The public defender's office could not represent him due to a conflict of interest from a related case. The trial court therefore appointed a lawyer to defend the officer. The lawyer is only three years out of law school. The lawyer practices criminal defense, but he has never handled a murder case before. For which of the following reasons may the lawyer decline the court appointment? A Based on what he has read in the newspapers, he sincerely believes that the officer is guilty. B He has no experience in the defense of a murder case. C He is of the same race as the teenage victim, and he is in sympathy with the plight of young gang members. D He recently was diagnosed with severe depression, which is affecting his ability to handle his existing caseload.

D He recently was diagnosed with severe depression, which is affecting his ability to handle his existing caseload. A lawyer is subject to discipline for trying to avoid a court appointment without good cause. [ABA Model Rule 6.2] (D) is good cause because the lawyer's severe depression is preventing him from handling his existing caseload competently. [See comment 2 to ABA Model Rule 6.2—good cause exists if the lawyer could not handle the matter competently; and see ABA Model Rule 1.16(a)(2)—requiring withdrawal when the lawyer's physical or mental condition materially impairs the lawyer's ability to represent the client] (A) is not good cause because a lawyer's belief that the defendant is guilty is not a sufficient reason to turn down a court appointment. Competent defense of a murder case certainly does not require a defense lawyer to believe in the client's innocence. (B) is not good cause because the facts state that the lawyer is three years out of law school and practices criminal defense law. That indicates that his training in criminal law and procedure is recent, and that he knows how to defend a criminal case, even though he has not handled a murder case before. Thus, the lawyer cannot claim lack of competence as an excuse for turning down the appointment. [See Hazard & Hodes, �51.3] ABA Model Rule 6.2(c) recognizes that a lawyer may turn down an appointment if the client or cause is so repugnant to him as to interfere in the lawyer-client relationship. Neither the lawyer's race nor his sympathy for young people who get involved with gangs should be regarded, without more, as likely to interfere with the lawyer's ability to represent the police officer competently. Thus, (C) is not good cause.

A lawyer agreed to represent a wife on an hourly fee basis in securing a divorce from her husband. The husband is also represented by an attorney. Despite repeated warnings by her attorney, the wife kept pestering her attorney with telephone calls and office visits concerning inconsequential details and trifling personal complaints. When the wife was unable to contact her own attorney on the phone or in person, she would telephone her husband's attorney, and try to put her questions and complaints to him. The husband's attorney always refused to talk to his client's wife. The wife's attorney repeatedly told her not to contact her husband's attorney, but to no avail. Finally, the wife's attorney told the wife that she would withdraw unless the wife changed her ways, but the wife did not do so. The wife's attorney withdrew and sent the wife a fee bill for the total number of hours she had spent on the case. The wife refused to pay the bill, and after futile efforts to settle the matter, the wife's attorney sued her to collect the fee, and stated that she would be holding the file until her bill was paid. Which of the following propositions is not true? A It was proper for the wife's attorney to withdraw. B It was proper for the husband's attorney to refuse to talk with the wife on the phone. C It was proper for the wife's attorney to bill the wife for the total amount of time she spent on the case. D It was proper for the wife's attorney to hold the wife's file until paid.

D It was proper for the wife's attorney to hold the wife's file until paid. It was improper for the lawyer to hold the client's file. (A) is true because one of the grounds for permissible withdrawal is that the client has made the lawyer's task unreasonably difficult (e.g., the client will not cooperate with the lawyer). [ABA Model Rule 1.16(b)(6)] (B) is true because a lawyer must not communicate about a matter with a person who is represented by another lawyer in the matter, unless that other lawyer consents. [ABA Model Rule 4.2] (C) is true because there is no reason in this case for the wife's attorney to charge the wife less than the full amount (assuming, of course, that the fee is reasonable in the circumstances). However, a lawyer has a duty to turn over the papers to the client. [See ABA Model Rule 1.16(d)]

A steel company merged with an iron corporation. The state attorney general sued the steel company and the iron corporation in federal court to enjoin the merger, alleging that it was in violation of the federal antitrust laws. The federal district judge enjoined the merger, and the steel company appealed the judge's decision. The steel company's lawyer, doing the legal research for the appeal, found a recent merger decision rendered by the Federal Trade Commission ("FTC") that is directly adverse to the steel company's position. FTC decisions do not control in the United States Courts of Appeal, but they are persuasive. The attorney general failed to cite the FTC decision. Must the steel company's attorney disclose it to the court? A Yes, because it is persuasive authority. B Yes, because the FTC decision is directly adverse to the steel company's position. C No, because an attorney has no obligation to volunteer facts harmful to his client's case. D No, because the court of appeals is not obliged to follow the FTC ruling.

D No, because the court of appeals is not obliged to follow the FTC ruling. The steel company's lawyer need not disclose the FTC ruling because the court need not follow the decision. An attorney can be disciplined for failing to cite the court to legal authority that is "directly adverse" to the client's position and is from the "controlling jurisdiction." [ABA Model Rule 3.3(a)(2)] Although the decision is directly adverse to the steel company's position, the facts state that FTC decisions do not control in the United States Courts of Appeal. Thus, the decision is not from the "controlling jurisdiction." (A) is wrong because in order to invoke the disclosure rule, the decision must be from a controlling jurisdiction, not merely persuasive. (B) is wrong because it states only one portion of the test that triggers the disclosure rule. While true that the decision is adverse, it is not from a controlling jurisdiction, and thus need not be revealed. (C) is wrong because although the Rule as stated is true, in this case we are not concerned with harmful facts. The issue is whether the steel company's attorney must reveal harmful law.

An attorney has organized his law practice as a professional corporation. The attorney is the sole shareholder. The sign on the office door states: "Professional Corporation— Attorney at Law Corporate and Business Law, Torts and Domestic Relations" The attorney has one lawyer-employee, who was admitted to practice two years ago. The attorney pays his employee a modest monthly salary plus 60% of the fees collected in cases that the employee handles by herself. The attorney has a general business practice and is not a certified specialist in any practice area. When a client needs representation in a tort or domestic relations matter, the attorney turns the case over to his lawyer-employee. When the attorney turns a case over to the employee, he provides general guidance and is available to answer any questions she may have, but he does not supervise every step she takes. Is the attorney subject to discipline? A Yes, because he splits fees with his employee in matters she handles by herself. B Yes, because he does not closely supervise the work done by his employee. C No, but he should change his sign to show the fields of practice that he personally handles. D No, because the employee is a lawyer-employee of the attorney.

D No, because the employee is a lawyer-employee of the attorney. The attorney is not subject to discipline. Because the employee is a lawyer-employee of the attorney, she is regarded as being "in the same firm." She and the attorney are thus allowed to split fees without complying with the rules that govern fee splits between lawyers who are not in the same firm. [ABA Model Rule 1.5(e)] (A) is wrong for the reason just stated. (B) is wrong because no Rule requires the attorney to supervise the lawyer-employee at every turn, so long as he takes reasonable steps to assure that she performs her work competently and otherwise within the bounds of legal ethics. [See ABA Model Rule 5.1—supervisory duties of lawyers within a firm] (C) is wrong; a law firm may state particular fields of law that the firm's attorneys handle. [ABA Model Rule 7.2, comment 9]

An attorney represented a landlord in a dispute with her longtime tenant, who had recently decided not to renew his lease. The landlord wanted to retain the security deposit to pay for extensive damage to the carpeting, while the tenant insisted that the damage was normal wear and tear. The attorney and the tenant's lawyer negotiated for days, but neither party would settle for less than two-thirds of the security deposit. Finally, the landlord telephoned the attorney and said: "The tenant asked if I want to talk about the security deposit. We've known each other for years, and I think we might have better luck if we work things out ourselves." The attorney encouraged the landlord to talk with the tenant if she thought it would help, but advised her not to finalize any agreement until both parties could consult with their respective counsel. The landlord and tenant had a productive discussion. The following day, the tenant's lawyer called the attorney and said the tenant would accept one-half of the security deposit. The attorney communicated the offer to the landlord, who agreed and returned the funds to the tenant. Is the attorney subject to discipline? A Yes, because the attorney encouraged the landlord to speak to the tenant without obtaining the consent of the tenant's lawyer. B Yes, because the attorney encouraged the landlord to speak to the tenant without notifying the tenant's lawyer. C No, because the tenant initiated the conversation with the landlord. D No, because the landlord and tenant spoke to each other directly.

D No, because the landlord and tenant spoke to each other directly. An attorney who represents a client in a matter must not communicate about the matter with a person the attorney knows is represented by counsel, unless that person's counsel consents or the communication is authorized by law or a court order. [ABA Model Rule 4.2] Here, however, the attorney did not communicate with the tenant. Rather, the attorney told the landlord to talk with the tenant if she thought it would be helpful, and even took care to make sure that the parties would not finalize any agreement without consulting their respective counsel. Although a lawyer must not use an agent to do what the lawyer is prohibited from doing [ABA Model Rule 8.4(a)], that was not the case here. Comment 4 to Rule 4.2 states that parties may communicate with each other directly, as the landlord and tenant did, and an attorney is not prohibited from advising a client concerning a communication that the client is legally entitled to make. Because the attorney was not required to obtain the consent of the tenant's lawyer in this situation, (A) is incorrect. Similarly, (B) and (C) are incorrect because the no-contact rule does not apply to these facts. Even if it did apply, these choices do not accurately reflect Rule 4.2. The attorney must obtain consent from the represented person's lawyer before communicating with the person about the matter; notifying the lawyer is not sufficient. Additionally, it is immaterial whether a represented person initiates or consents to a prohibited communication.

A lawyer regularly represented an older client in matters relating to the investment of the client's considerable wealth. The client told the lawyer that he wanted to put $500,000 into a sound, income-producing investment. The lawyer suggested that the two of them pool their money and talent and buy an apartment house. The lawyer would put up $75,000 and do the legal work, and the client would put up $500,000 and serve as the live-in manager of the apartment house. The client enthusiastically agreed to the arrangement and told the lawyer to draw up the papers. The lawyer drafted an agreement between himself and the client, negotiated the purchase of the apartment house, and drafted a deed from the seller to himself and the client as joint tenants with right of survivorship. The lawyer gave the client a carefully written explanation of the terms of the transaction, but he forgot to explain the significance of the joint tenancy, i.e., that upon the death of one joint tenant, the property would pass automatically to the other joint tenant. The lawyer advised the client, in the writing explaining the terms of the transaction, to have an outside lawyer look over the transaction, and he also urged him orally to do so. However, the client said that he trusted the lawyer and signed all of the papers without further ado. The lawyer and the client operated the apartment house successfully for several years, until the client died. The executor of the client's estate sued the lawyer to have the apartment house declared part of the client's estate, but the court concluded that the joint tenancy created a gift to the lawyer, effective on the client's death. Were the lawyer's actions proper? A Yes, because the court concluded that the joint tenancy created a gift from the client to the lawyer. B Yes, because the lawyer might have died first, thus bestowing a gift on the client. C No, because the lawyer entered into a business transaction with the client. D No, because the lawyer drafted the deed that bestowed a substantial gift on himself.

D No, because the lawyer drafted the deed that bestowed a substantial gift on himself. The lawyer's actions were not proper because a lawyer may not draft a legal instrument for a client that gives a substantial gift to the lawyer, unless the client is a relative of the lawyer. [ABA Model Rule 1.8(c)] The deed creating the joint tenancy bestows a substantial gift on the lawyer, particularly in light of the high likelihood that the client would die first. The transaction is also particularly suspect in light of the lawyer's failure to explain the joint tenancy to the client. [See ABA Model Rule 1.8(a)—business transaction with client requires full disclosure of interest granted lawyer] (A) is wrong because the court's determination does not free the lawyer from discipline for drafting the deed that bestowed the gift on him. (B) is wrong because the right of survivorship is a substantial gift regardless of who actually dies first; thus, it was not proper for the lawyer to draft the document. (C) is wrong because it is overbroad. A lawyer may enter into a business transaction with a client, provided he follows certain safeguards. Here, for example, had the lawyer met all of the requirements of ABA Model Rule 1.8(a), the transaction would have been a proper business transaction. However, the transaction still would have been improper because the lawyer drafted the deed.

A man alleges that a very wealthy actor punched him in the face. He contacted a lawyer about representing him in a civil action against the actor. After several lengthy discussions with the lawyer about the merits of the case, the man decided to employ another attorney instead. The actor was later charged with criminal assault in connection with this incident, and the trial was televised. As the lawyer was watching the trial, she was astonished when the man testified to facts that the lawyer knew from their previous discussions to be false. The lawyer sent a letter with a messenger over to the court to notify the court that the man had perjured himself. Were the lawyer's actions proper? A Yes, because her actions were necessary to prevent the man from perpetrating a fraud on the court. B Yes, because the man committed a criminal act by testifying falsely. C No, unless she sent copies of the letter to the prosecution and defense attorneys and they are given an opportunity to respond. D No, because the lawyer's information was gained during her discussions with the man.

D No, because the lawyer's information was gained during her discussions with the man. The lawyer's actions were not proper because her information was gained from a confidential lawyer-client communication. [See ABA Model Rule 1.6] It was both privileged and confidential, and could be disclosed only if one of the exceptions to the duty of confidentiality applies. None of the exceptions applies here. The fact that the man did not hire the lawyer to represent him does not affect her duty of confidentiality; he was seeking legal advice and representation when he spoke to her. [See ABA Model Rule 1.18(b)] (A) is wrong because there is no exception to the duty of confidentiality to prevent a fraud on the court when the lawyer is not appearing before the court. (B) is wrong because this too does not fit within any exception to the duty of nondisclosure. A lawyer may reveal confidential information to the extent she reasonably believes necessary to prevent, mitigate, or rectify substantial injury to the financial interests of another that is reasonably certain to result or has resulted from the client's commission of a crime or fraud, if the client has used the lawyer's services in furtherance of such crime or fraud. [ABA Model Rule 1.6(b)(3)] Here, the facts do not indicate that the man used the lawyer's services in furtherance of his perjurious testimony. Thus, although the perjury could result in substantial injury to the actor's financial interests, the lawyer is not permitted to reveal the confidential information. (C) is wrong because it states the rule for an ex parte communication to the judge by one of the parties. Here, the lawyer does not represent one of the parties. Furthermore, the information is confidential and cannot be disclosed to the disadvantage of the client regardless of whether the lawyer sends copies to all parties.

A patent attorney focuses her practice on patents that involve genetically engineered medicines. Representatives of a bioengineering firm had a preliminary conversation with the attorney about representing the bioengineering firm in a patent infringement action against a pharmaceutical corporation. The attorney had never represented either company previously. The bioengineering firm's representatives talked to the attorney for more than an hour about the bioengineering firm's patent and about the pharmaceutical corporation's supposedly infringing product. This conversation covered only public information, nothing confidential. The bioengineering firm's representatives detected a distinct lack of enthusiasm from the attorney, and they ended the conversation cordially but without hiring her. In due course, the bioengineering firm hired a different patent attorney and sued the pharmaceutical corporation for patent infringement. The pharmaceutical corporation hired the attorney as defense counsel in the infringement case. The bioengineering firm's attorney promptly made a motion in the trial court to disqualify the attorney because of her earlier conversation with the bioengineering firm's representatives. Is the attorney subject to disqualification? A Yes, because the bioengineering firm had previously consulted the attorney on the same matter. B Yes, because the infringement suit is substantially related to the earlier conversation between the attorney and the bioengineering firm's representatives. C No, because the bioengineering firm was never the attorney's client. D No, because the prior conversation between the attorney and the bioengineering firm's representatives did not involve confidential information.

D No, because the prior conversation between the attorney and the bioengineering firm's representatives did not involve confidential information. ABA Model Rule 1.18 provides that a lawyer must not use or reveal confidential information of a prospective client. [See also ABA Model Rule 1.6] Here, the information communicated to the patent attorney was not confidential; thus, the patent attorney's undertaking representation of the pharmaceutical corporation did not create a concurrent conflict of interest—there is no significant risk that the representation of the pharmaceutical corporation would be materially limited by the patent attorney's responsibilities to the bioengineering firm. Consequently, the patent attorney may continue to represent the pharmaceutical corporation. (A) is wrong because it does not matter that the bioengineering firm previously consulted the patent attorney on the same matter if the patent attorney did not obtain any confidential information that would limit her representation of the pharmaceutical corporation, and she does not breach any duty owed to the bioengineering firm. (B) is wrong for the same reason as (A)—even if the infringement suit is substantially related to the patent attorney's conversation with the bioengineering firm's representatives, unless the patent attorney obtained confidential information from the bioengineering firm, she may represent the pharmaceutical corporation. (C) is not as good as (D) because (C) is general, while (D) is specifically on point. Here, the patent attorney did not get any material confidential information from the bioengineering firm's representatives during the preliminary conversation, but if she had, she would be subject to disqualification as defense counsel, even though the bioengineering firm never became the patent attorney's actual client.

For the past five years, an attorney has represented an art dealer in the sale of many valuable paintings. One of the major transactions occurred three years ago, when the art dealer sold a landscape purportedly painted by Vincent van Gogh to an art museum for $23 million. The museum subsequently resold the painting for $35 million. Now the art dealer has asked the attorney to do the legal work in connection with the sale of another landscape, also a purported van Gogh. The proposed purchase price is $12 million, and the prospective purchaser is a wealthy television personality who knows nothing about art. During a confidential conversation in the attorney's office, the attorney asked the art dealer if he had appraisal letters certifying the painting as a genuine van Gogh. The art dealer replied that he indeed had letters—letters he had forged himself—and that he had also forged the letters for the purported van Gogh sold to the art museum. When the attorney inquired further, the art dealer told him in confidence that both of the purported van Gogh paintings were in fact counterfeits created by a clever art student. Which of the following must the attorney do at this point? A Report the art dealer to the law enforcement authorities. B Warn the prospective purchaser about the proposed sale. C Inform the art museum of the truth about the first painting. D Refuse to represent the art dealer in the present transaction.

D Refuse to represent the art dealer in the present transaction. The attorney must refuse to represent the art dealer in the present transaction. Note that the call of the question asks what the attorney must do, not what he would be allowed to do. ABA Model Rule 1.2 states that a lawyer must not counsel or assist a client in conduct that the lawyer knows is criminal or fraudulent. Here, the attorney must refuse to represent the art dealer in the sale of the painting to the television personality because the sale would be fraudulent—the attorney knows that the painting is not a van Gogh as the art dealer represented to the television personality. ABA Model Rule 1.6(b) permits a lawyer to reveal a client's confidential information to the extent necessary to prevent the client from committing a crime or fraud that would result in substantial financial harm to a person, if the client is using or has used the lawyer's services to further the crime or fraud. A lawyer also may reveal confidential information if the client has already acted and the disclosure will mitigate the consequent financial harm. Thus, the attorney is permitted to disclose information to law enforcement authorities in order to prevent harm to the television personality and possibly mitigate subsequent financial harm caused by the first transaction to the art museum; however, the attorney is not required to do so. The same holds true regarding the attorney's warning the television personality about the proposed sale. Consequently, (A) and (B) are wrong. (C) is also wrong because even if it can be argued that the attorney's revealing information about the fraudulent sale to the museum would mitigate subsequent financial harm, the attorney is not required to reveal the art dealer's confidential information.

A law student is applying for admission to the State A Bar. When the law student was in high school, he and his parents lived in State B. His next door neighbor was an attorney admitted to practice in State B, but not in State A. The attorney knew that during the law student's senior year in high school, he was convicted of burglarizing a liquor store. After serving his sentence, the law student went to college and later to law school. The attorney has had no contact with the law student since his high school years, and as far as she knows, the law student has not done anything since high school that would reflect badly on his character. The Bar of State A sent the attorney a routine questionnaire, asking a series of questions about the law student's character. The attorney does not know whether the law student disclosed the burglary conviction on his bar application, and she does not know where to contact him to find out. Which of the following would be a proper response to the questionnaire? A She should not respond at all because she has no relevant information to provide. B She should not respond at all because as a State B lawyer she is not obligated to provide information to the Bar of State A. C She should not mention the law student's burglary conviction in her response unless she first contacts him and obtains his permission to do so. D She should state what she knows about the law student, including mention of his burglary conviction.

D She should state what she knows about the law student, including mention of his burglary conviction. The attorney should state what she knows about the law student, including mentioning his burglary conviction. An attorney who is properly asked for information about a bar applicant's character has a duty to respond and to do so accurately. [ABA Model Rule 8.1] (A) is wrong because the attorney does have relevant information—she knows about the law student's burglary conviction, and that is relevant to (but certainly not conclusive of) the inquiry about his character. (B) is wrong because an attorney's duty to provide bar applicant information is not confined to the state in which the attorney practices. (C) is wrong because the burglary conviction is relevant to the character inquiry, and nothing indicates that the attorney learned about it in confidence.

A plaintiff brought a civil action to recover damages for personal injuries he suffered as the victim of alleged police brutality inflicted by three defendant police officers. The trial was widely reported by the media. The jury returned a verdict in favor of the plaintiff and against the three police officers for $500 million. When the trial judge received the verdict, he was shocked by the size of the award. Before dismissing the jurors, the judge told the jurors that when they were sworn in, they had promised that they would deliver a verdict based on the evidence and that they would not be swayed by passion or prejudice. The judge further admonished the jurors that they had failed in those duties, that they had made a mockery of justice, and that they should be ashamed of themselves. He then dismissed the jury, and the defense lawyers renewed their motion for judgment as a matter of law and, alternatively, moved for a new trial. The judge announced that he would rule on the motions the following Monday at 10 a.m. in open court. The press reports of the verdict and the judge's comments to the jury created a great public tumult in the city where the case was tried. On the following Monday, the courtroom was jammed with reporters. Primarily for the purpose of educating the reporters, the judge first gave a detailed explanation of the legal requirements for granting a renewed motion for judgment as a matter of law and for granting a new trial motion. He then granted the renewed motion for judgment as a matter of law and, alternatively, the motion for a new trial. Were the judge's actions proper? A Both the statements to the jury and the communication with the reporters were proper. B Neither the communication with the reporters nor the statements to the jury were proper. C The statements to the jury were proper, but the communication with the reporters was not. D The communication with the reporters was proper, but the statements to the jury were not.

D The communication with the reporters was proper, but the statements to the jury were not. The communication with the reporters was proper, but the statements made to the jury were not. A judge should not "commend or criticize jurors for their verdict other than in a court order or opinion in a proceeding." A judge's commendation or criticism may impair a juror's ability to be impartial in a subsequent case. [CJC Rule 2.8(C)] The judge's statements to the jury clearly violate this Rule; thus, (A) and (C) are incorrect. Although judges should not comment on pending cases in a manner that might interfere with fairness, judges are not prohibited from making public statements in the course of their official duties or from explaining for public information the procedures of the court. [CJC Rule 2.10(D)] The judge's explanation of the motions to the reporters was an appropriate way to inform the public of the meaning and significance of the judge's decision to reject the $500 million verdict; thus, (B) and (C) are incorrect.

When a lawyer is representing a corporate client, which of these is NOT required for the lawyer's communications with a lower level corporate employee to be protected by the attorney-client privilege? A The employee communicates with the lawyer at the direction of the employee's superior B The communication concerns a subject within the scope of the employee's duties to act for the corporation C The employee knows that the purpose of the communication is to obtain legal advice for the corporation D The employee invokes the attorney-client privilege on behalf of the corporation

D The employee invokes the attorney-client privilege on behalf of the corporation There is no need for the employee to explicitly invoke the attorney-client privilege for the privilege to apply. When representing a corporate client, conversations between a lawyer and a high ranking corporate official are covered by the client attorney privilege. In addition, the privilege also covers communications between the lawyer and a lower level corporate employee if the following conditions are met: (i) the employee communicates with the lawyer at the direction of the employee's superior; (ii) the employee knows that the purpose of the communication is to obtain legal advice for the corporation; and (iii) the communication concerns a subject within the scope of the employee's duties to act for the corporation.

A lawyer was a widely admired, highly compensated trial attorney in solo practice. He represented clients in all types of civil and criminal litigation, mostly in high-profile cases that drew a lot of media attention. The governor of the state where the lawyer practiced had been harshly criticized for appointing appellate judges who lacked significant experience as trial counsel. Hoping to silence his critics, the governor appointed the lawyer to serve out the remaining seven years of a recently deceased supreme court justice's 12-year term. After the seven years, the lawyer can run for election to a new 12-year term. Before taking the oath as judge, the lawyer sold his entire law practice—books, client files, office lease, furniture, and goodwill—to another attorney. The lawyer gave appropriate advance notice to the clients, and the purchasing attorney covenanted that he would not raise their legal fees. A few years later, one of the cases that the lawyer transferred to the purchasing attorney came before the state supreme court on appeal. Which of the following propositions is false? A The lawyer's sale of his law practice was proper. B The purchaser's covenant not to increase the fees paid by the lawyer's clients was proper. C The lawyer must disqualify himself from the case involving his former client. D The lawyer may participate in the decision of the case involving his former client, provided that all of the other supreme court justices give their informed consent.

D The lawyer may participate in the decision of the case involving his former client, provided that all of the other supreme court justices give their informed consent. (D) is false. CJC Rule 2.11(C) explains remittal of a judge's disqualification, and remittal requires the agreement of all of the parties and their lawyers, not the other justices. (A) is true. ABA Model Rule 1.17 permits the sale of an entire law practice, or an area of practice, subject to some conditions, all of which are met here. (B) is true. ABA Model Rule 1.17(d) provides that when a practice is sold, the fees charged to the clients cannot be increased by reason of the sale. (C) is true. CJC Rule 2.11(A)(6)(a) says that a judge must disqualify himself if he previously served as a lawyer in the matter.

An author wrote a best-selling novel based on the life and crimes of John Dillinger, the famous bank robber. The author sold the movie rights to a film producer, who promised to pay the author a lump-sum royalty of $5 million upon the release of the movie. After the producer hired an actor to play the lead role and made other expensive preparations for filming, the author repudiated the contract. The producer hired a lawyer to sue the author for a declaratory judgment that the contract was valid and enforceable. At the producer's request, the lawyer agreed to do the legal work on a contingent fee basis: If the producer wins, the lawyer will be paid 1.75% of the gross receipts from the movie, but if the producer loses, the lawyer will be paid nothing. The producer and the lawyer entered into a written fee agreement that contains all the details required by the rules of legal ethics. Which of the following statements is true? A The lawyer is subject to discipline for entering into a publication rights contract with his client. B The lawyer is subject to discipline for acquiring a personal interest in the subject of the litigation. C The lawyer's fee agreement is proper, but only if the author gives informed consent. D The lawyer's fee agreement is proper, even though it gives the lawyer a personal interest in the subject of the litigation.

D The lawyer's fee agreement is proper, even though it gives the lawyer a personal interest in the subject of the litigation. The lawyer's fee agreement is proper even though it gives the lawyer a personal interest in the subject of the litigation. The lawyer has acquired a personal interest in the movie, which is in one sense the real subject of the litigation. However, the rule against acquiring a personal interest in the subject of litigation has an exception that allows a lawyer to represent a client for a contingent fee. [ABA Model Rule 1.8(i)(2)] (A) is wrong because the rule on literary rights contracts covers only literary works based in substantial part on information relating to the representation. Here, the movie concerns John Dillinger, not the producer. [See ABA Model Rule 1.8(d)] (B) is wrong because, as discussed above, a lawyer may acquire an interest in the subject matter of the litigation in the form of a contingent fee. (C) is wrong because the fee agreement between the producer and the lawyer does not require the author's informed consent. The author is not a current or former client of the lawyer, and the author has no apparent interest that would cause the lawyer a conflict and force him to disclose and explain the material risks and available alternatives and obtain the author's consent.

A prospective client comes to a law office seeking a lawyer to defend him in a civil action for aggravated assault and battery. A lawyer agrees to talk preliminarily with the client, just to obtain enough background information to decide whether she can defend him. The client explains that he has an alcohol problem; indeed, he gets roaring drunk about three nights a week. On the night in question, the client said that a loud-mouthed stranger in his neighborhood tavern made a derogatory comment about the client's favorite basketball team. The client responded by "tapping" the stranger over the head with a pool cue, not once but four times. At that point, the lawyer suddenly realizes that the client must be the rotten husband in the hotly disputed divorce and child custody case in which her law partner is representing the aggrieved wife. The lawyer stops the client and tells him that she cannot defend him in the assault and battery case because of her partner's work for the client's wife. Which of the following is true? A The partner must withdraw from representing the wife because the lawyer has received confidential information from the client that would be harmful to the client if used in the divorce and child custody case. B It would be proper for the partner to represent the wife and for the lawyer to represent the client in the assault and battery case because the two matters are not substantially related. C The partner may continue representing the wife, but only if the wife gives informed consent, confirmed in writing. D The partner may continue representing the wife if the lawyer is screened off from participation in the case and obtains no part of the fee in the case, and if the firm promptly sends the client written notice of the situation.

D The partner may continue representing the wife if the lawyer is screened off from participation in the case and obtains no part of the fee in the case, and if the firm promptly sends the client written notice of the situation. This question is governed by ABA Model Rule 1.18, which concerns duties to a prospective client. The information that the lawyer obtained about the prospective client's alcohol abuse and his violent response to the stranger's comment could be harmful to the prospective client if the wife uses it to help prove that the couple should be divorced and that the prospective client should not be given custody of their children. Therefore, the lawyer herself could not represent the wife in the divorce and child custody case. [See ABA Model Rule 1.18(c)] The lawyer's disqualification is imputed to her law partner. [Id.] However, if the conditions mentioned in (D) are satisfied, then her law partner may continue representing the wife. [See ABA Model Rule 1.18(d)(2)] (C) is wrong because it calls for informed consent by the wife only. ABA Model Rule 1.18(d)(1) would require informed consent, confirmed in writing, from both the prospective client and the wife. (B) is wrong because the two matters are "substantially related" [ABA Model Rule 1.18(c)] in that use of the information that the prospective client disclosed to the lawyer could be harmful to the prospective client if used in the divorce and child custody case. (A) is wrong because it ignores the two possible ways that would allow the law partner to continue representing the wife. [ABA Model Rule 1.18(d)(1), (2)]

When must a lawyer report another lawyer's actions to the appropriate professional authority? A When the lawyer knows that the other lawyer has violated the Rules of Professional Conduct in any way B When the lawyer suspects that the other lawyer has violated the Rules of Professional Conduct in any way C When the lawyer suspects that the other lawyer has violated the Rules of Professional Conduct in such a way that it raises a substantial question as to that lawyer's honesty, trustworthiness, or fitness as a lawyer D When the lawyer knows that the other lawyer has violated the Rules of Professional Conduct in such a way that it raises a substantial question as to that lawyer's honesty, trustworthiness, or fitness as a lawyer

D When the lawyer knows that the other lawyer has violated the Rules of Professional Conduct in such a way that it raises a substantial question as to that lawyer's honesty, trustworthiness, or fitness as a lawyer A lawyer who knows that another lawyer has violated the Rules of Professional Conduct in such a way that it raises a substantial question as to that lawyer's honesty, trustworthiness, or fitness as a lawyer must report the violation to the appropriate professional authority. A lawyer who fails to report this type of misconduct is herself subject to discipline for violating the rule requiring disclosure. A lawyer may choose to report suspected misconduct, but she is not required to do so. Similarly, a lawyer is not required to make a report whenever another lawyer has violated a rule in any way. She may decline to report another lawyer's violation if it concerns an arcane rule and does not indicate that the other lawyer is dishonest, untrustworthy, or unfit to practice law. Examples of violations that may not raise a substantial question as to a lawyer's fitness to practice include some types of improper solicitation and the improper use of nonlawyer assistants.

An elderly widower has one living child, a daughter. The widower's main asset is a 51% partnership interest in a wealthy real estate syndicate that owns and operates mobile home parks throughout the state. The daughter's husband is an attorney. One of the husband's regular clients asks the husband to represent him in negotiating the sale of 3,000 acres of roadside property to the real estate syndicate. The real estate syndicate is represented by its own lawyer in the matter. May the husband represent his regular client in a sale with the real estate syndicate? A No, even if the client gives informed consent, confirmed in writing. B No, because to do so would create an appearance of impropriety. C Yes, because the husband has no significant personal interest in the real estate syndicate. D Yes, but only if the client gives informed consent, confirmed in writing.

D Yes, but only if the client gives informed consent, confirmed in writing. The husband may represent his regular client if the client gives informed consent, confirmed in writing, to the representation. His wife is likely to inherit her father's interest in the real estate syndicate. That gives the husband a personal interest in the real estate syndicate, albeit an attenuated interest. If the husband is to represent his regular client in selling land to the real estate syndicate, he must first disclose his personal interest to the client. If the client gives informed consent, confirmed in writing, then the husband may represent the client. [ABA Model Rule 1.7(b)] (A) is wrong because informed, written consent will solve the potential conflict of interest. (B) is wrong because informed consent, confirmed in writing, will solve the conflict problem. Furthermore, the "appearance of impropriety" is not a basis for discipline or disqualification under the ABA Model Rules. This is an outdated concept from the old ABA Model Code. (C) is wrong because the husband's personal interest is significant, even though it is remote.

Two years ago, an attorney represented his client when he sold his property. Unbeknownst to the attorney, the client made some fraudulent statements to the buyer about the value of some mineral deposits on the property. The buyer recently discovered the fraud and is now in the attorney's office threatening to immediately file a civil fraud suit against both the client and the attorney. The buyer accuses the attorney of engineering the fraud and helping his client carry it out. The only way that the attorney can convince the buyer that he had no part in the fraud is to tell the buyer a fact that the client disclosed to him in the deepest confidence when he was working on the property transaction. May the attorney disclose the fact without the consent of the client? A No, if doing so will harm the client. B No, because doing so would breach his duty of confidentiality to the client. C Yes, but only after the buyer files the civil fraud suit against him. D Yes, even if doing so will subject his client to civil or criminal liability.

D Yes, even if doing so will subject his client to civil or criminal liability. The attorney may reveal the confidence even if doing so will subject his client to civil or criminal liability. A lawyer may disclose a client's confidence "to establish a defense to a criminal charge or civil claim against the lawyer based upon conduct in which the client was involved . . . ." [ABA Model Rule 1.6(b)(5)] Although the lawyer must wait until the assertion of misconduct arises, he need not await the filing of a formal charge or complaint. The lawyer may defend himself by responding directly to a third party who has made such an assertion. [See Restatement �64, comment c] (A) is wrong because the lawyer may disclose the fact even if doing so harms the client. (C) is wrong because the lawyer need not wait for the complaint to be filed, as explained above. (B) is wrong because it ignores the self-protection exception to the general rule of confidentiality.

A solo practitioner is one of only three lawyers in a small town. The solo practitioner is presently defending a client in a criminal action for assault and battery. This morning one of the solo practitioner's regular clients, a gas and grocery store, asked the solo practitioner to sue the same client to recover a past due amount on a gasoline and grocery charge account. Would it be proper for the solo practitioner to represent the gas and grocery store in the charge account case? A No, because it is presumed that a lawyer obtains confidential information in the course of representing a client. B No, unless the other two lawyers in town are disqualified from representing the gas and grocery store. C Yes, because there is no substantial relationship between the charge account case and the assault and battery case. D Yes, if both the client and the gas and grocery store consent after full disclosure of the conflict, and such consent is confirmed in writing.

D Yes, if both the client and the gas and grocery store consent after full disclosure of the conflict, and such consent is confirmed in writing. It would not be proper for the solo practitioner to represent the grocery store unless both the client and the grocery store give informed consent, confirmed in writing, to the representation. A lawyer who is presently representing a client in one litigation matter should not simultaneously oppose that client in a different litigation matter, even if the two matters are unrelated. [ABA Model Rule 1.7(a) and comment 6] The purpose of the rule is to avoid putting the client into the difficult position of treating the lawyer simultaneously as friend and foe. The conflict can be solved only by informed consent, confirmed in writing, from both the client and the grocery store. (A) is wrong because the rule prohibiting the representation applies regardless of whether the solo practitioner has obtained relevant confidential information. (B) is wrong because the conflict is not obviated by the shortage of legal talent in the small town. (C) is wrong because the Rule applies even if the two cases are unrelated.

A plaintiff, represented by his lawyer, brought suit in federal district court against a pest control company and nine chemical companies for physical and emotional injuries the plaintiff suffered after accidentally inhaling cockroach spray emanating from an apartment that had recently been fumigated by the pest control company. The lawyer's theory for suing the nine chemical companies was that the pest control company had probably purchased its cockroach spray from at least one of the nine chemical companies. A large law firm represented one of the nine chemical company defendants. By using depositions and document demands early in the discovery phase of the case, the law firm established that the chemical company it represented had never at any time sold any type of chemical to the pest control company. The law firm then moved for summary judgment as to its client. The plaintiff's lawyer offered no substantive response to that motion, but rather filed a countermotion to disqualify the law firm on the ground that the firm was biased against the plaintiff. The trial judge denied the motion to disqualify the law firm and granted the chemical company's summary judgment motion, whereupon the plaintiff's lawyer immediately moved for a rehearing, moved to stay the trial judge's two orders, and moved to disqualify the trial judge for bias and prejudice against the plaintiff and in favor of the defendant chemical company, the nature of the bias and prejudice being unspecified. The disposition of these motions consumed an entire year, due to the lawyer's obstreperousness and his repeated requests for postponements and extensions of time. Meanwhile, the law firm had to stay actively involved in the case to protect the chemical company's position. This year-long ordeal ended up costing the chemical company $14,500 in attorneys' fees and $6,750 in litigation costs. Is the plaintiff's lawyer subject to litigation sanction in the form of an order against the lawyer personally to pay the $14,500 in attorneys' fees and the $6,750 in litigation costs? A No, because the lawyer was representing his client zealously within the bounds of the law as he was required to do by the rules of legal ethics. B No, because litigation sanctions can be imposed only on parties to the litigation, not on their lawyers personally. C Yes, even if the lawyer was acting in good faith, mistakenly but genuinely believing in the validity of the legal positions he took. D Yes, provided that the chemical company can show that the lawyer either intentionally or recklessly took frivolous legal positions in order to harass the chemical company.

D Yes, provided that the chemical company can show that the lawyer either intentionally or recklessly took frivolous legal positions in order to harass the chemical company. The lawyer is subject to litigation sanction because Federal Rule of Civil Procedure 11 provides that sanctions can be imposed on a lawyer, firm, or party for filing a pleading, motion, or other paper merely to harass, delay, or multiply expenses for the opponent. [See also Hudson Motors Partnership v. Crest Leasing Enterprises, Inc., 845 F. Supp. 969 (E.D.N.Y. 1994)—federal courts have inherent power to sanction lawyers for frivolous legal positions to harass or delay; and see 28 U.S.C. �1927—court can impose sanctions on lawyers who knowingly or recklessly multiply proceedings unreasonably and vexatiously] (A) is incorrect because it is beyond the bounds of the law to harass an opponent with a legal position that the proponent knows is frivolous. (B) is incorrect because Rule 11 allows the court to sanction both the lawyers and the parties they represent. (C) is incorrect because lawyers are not subject to litigation sanction for taking legal positions that they, in good faith, believe to be meritorious.


Kaugnay na mga set ng pag-aaral

Chapter 15: Intraoperative Nursing Management

View Set